Test Bank – ACP: Cardiovascular Medicine

Test Bank – ACP: Cardiovascular Medicine

11. An 80-year-old man presents with dyspnea and is found to have CHF caused by systolic dysfunction. He also has mild renal insufficiency, with a creatinine level of 1.4.

Which of the following statements is true regarding ACE inhibitor therapy in this patient?
A. It is contraindicated because of his age
B. It is contraindicated because of his renal insufficiency
C. Angiotensin II receptor blockers are preferred for CHF in elderly patients
D. ACE inhibitor therapy can be started, provided it is accompanied by careful monitoring of his creatinine and potassium levels
E. ACE inhibitor therapy can be started but must be discontinued if his creatinine level rises above its current level

Key Concept/Objective: To be able to identify patients with CHF for whom ACE inhibitor therapy is indicated

All patients with CHF should be on an ACE inhibitor unless there is a very good reason not to use one. The only patients with CHF in whom an ACE inhibitor cannot be used are those in whom an ACE inhibitor causes hypersensitivity, hyperkalemia, or cough or those with bilateral renal artery stenosis. Neither this patient’s age nor his creatinine level is a contraindication to ACE inhibitor use (in fact, the improved forward flow may actually increase his glomerular filtration rate and improve his renal function). His creatinine and potassium levels should be checked 1 to 2 weeks after starting the medication to ensure that the creatinine level has not increased by more than 25% and that his potassium level is less than 5.0 mmol/L. Preliminary data suggested that angiotensin II receptor blockers were preferred over ACE inhibitors in older patients, but a larger trial failed to confirm these results. (Answer: D—ACE inhibitor therapy can be started, provided it is accompanied by careful monitoring of his creatinine and potassium levels)

12. A 70-year-old woman presents to the emergency department with acute pulmonary edema with evidence of myocardial ischemia on ECG. In spite of maximal medical management, she develops cardiogenic shock. A second ECG shows ST segment elevation of 3 mm in the precordial leads. She has no contraindications to thrombolytic therapy.

Which of the following statements regarding thrombolytic therapy is true?
A. Thrombolytic therapy is indicated, but direct revascularization is preferable if it can be obtained quickly
B. Thrombolytic therapy is contraindicated because of her age
C. Thrombolytic therapy is contraindicated because of the presence of cardiogenic shock
D. Thrombolytic therapy will establish antegrade coronary artery perfusion in 75% of cases
E. Thrombolytic therapy is contraindicated because of the risks of bleeding associated with it

Key Concept/Objective: To understand the indications for thrombolytic therapy in patients with cardiogenic shock caused by myocardial infarction

Patients who develop cardiogenic shock because of a myocardial infarction have dismal mortality rates; however, mortality can be lowered from 85% to less than 60% if flow can be reestablished in the infarct-related artery. Thrombolytic therapy is able to achieve this in about 50% of cases, making percutaneous angioplasty preferable; however, if angioplasty cannot be administered quickly or is not available, thrombolytic therapy is indicated. This patient’s age is not a contraindication to thrombolytic therapy, nor is the presence of cardiogenic shock (patients with cardiogenic shock benefit most from thrombolytic therapy). (Answer: A—Thrombolytic therapy is indicated, but direct revascularization is preferable if it can be obtained quickly)

For more information, see Winakur S, Jessup M: 1 Cardiovascular Medicine: II Heart Failure. ACP Medicine Online (www.acpmedicine.com). Dale DC, Federman DD, Eds. WebMD Inc., New York, November 2003

Hypertension

13. A 42-year-old white man presents to your office as a new patient. He has been in good health and has not seen a physician in many years. While attending a local health fair recently, the patient was told that he had high blood pressure, and he was advised to seek medical help. He seeks your advice about hypertension. His blood pressure is 145/95 mm Hg.

Which of the following general statements about hypertension is false?
A. Hypertension is the most common chronic disorder in the United States, affecting 24% of the adult population
B. Hypertension is a major risk factor for stroke, myocardial infarction, heart failure, chronic kidney disease, progressive atherosclerosis, and dementia
C. For a normotensive middle-aged person in the United States, the lifetime risk of developing hypertension approaches 90%
D. Hypertension is the third most common reason adults visit the doctor’s office; it is surpassed only by visits for upper respiratory complaints and low back pain

Key Concept/Objective: To understand the dramatic impact of hypertension on health care in the United States

Hypertension is the most common chronic disorder in the United States, affecting 24% of the adult population. In the year 2000, hypertension accounted for more than 1 million office visits to health care providers. The prevalence increases with age: for a normotensive middle-aged person in the United States, the lifetime risk of developing hypertension approaches 90%. With the increasing age of the population in most developed and developing societies, it seems safe to assume that hypertension will become steadily more widespread in the coming years. Hypertension is a major risk factor for stroke, myocardial infarction, heart failure, chronic kidney disease, progressive atherosclerosis, and dementia. Hypertension is the most common reason adults visit the doctor’s office. (Answer: D— Hypertension is the third most common reason adults visit the doctor’s office; it is surpassed only by visits for upper respiratory complaints and low back pain)

14. A 64-year-old black man presents to your office for routine follow-up care. You have treated him for many years for hypertension with a calcium channel blocker and a thiazide diuretic. His hypertension has been moderately well controlled with this regimen. He asks you whether having a home blood pressure monitor would be useful for his care.

Which of the following statements regarding ambulatory blood pressure monitoring (ABPM) is true?
A. ABPM is not useful in patients whose office blood pressure is normal and who have hypertensive target organ injury
B. ABPM is not a useful tool in the evaluation of suspected autonomic dysfunction in patients with orthostatic hypotension
C. Cross-sectional studies show that blood pressure averages obtained during office visits correlate better with the presence of target-organ injury (especially LVH) than those obtained with ABPM
D. ABPM is useful in establishing a diagnosis of white-coat hypertension

Key Concept/Objective: To understand the usefulness of ABPM

Cross-sectional studies show that blood pressure averages from ABPM correlate better with the presence of target-organ injury (especially left ventricular hypertrophy [LVH]) than office blood pressure averages. Also, prospective studies and population-based observational studies have shown that average blood pressure derived from ABPM predicts additional risk of cardiovascular (CV) events after adjustment for clinic or office blood pressure. ABPM is the best method to establish the presence of isolated clinic hypertension (socalled white-coat hypertension), which is defined as an elevation in BP that occurs only in the clinic setting, with normal BP in all other settings, in the absence of evidence of target-organ injury. Screening for white-coat hypertension is currently a reimbursable indication for ABPM by Medicare. Other uses for ABPM include assessment of hypotensive symptoms, episodic hypertension, and suspected autonomic dysfunction in patients with postural hypotension. ABPM is also useful in the evaluation of the occasional patient with hypertensive target-organ injury (LVH, stroke) whose office blood pressure is normal. (Answer: D—ABPM is useful in establishing a diagnosis of white-coat hypertension)

15. A 25-year-old black man presents to your office seeking to establish primary care. The patient has no complaints and denies any known medical history. His blood pressure is noted to be 185/115; otherwise, his physical examination is normal. After measuring the patient’s blood pressure a total of four times during two office visits, you diagnose hypertension.

Which of the following statements regarding the initial evaluation of hypertension in this patient is true?
A. A retinal examination should now be performed on this patient
B. Because this patient has been diagnosed with hypertension, obtaining a family history of hypertension or early CV disease is no longer useful
C. During the initial examination, only a single careful blood pressure measurement is needed
D. This patient has no findings consistent with secondary hypertension

Key Concept/Objective: To understand the importance of the initial evaluation of hypertension

Secondary hypertension may be suspected on finding features that are not consistent with essential hypertension. Such features include age at onset younger than 30 years or older than 50 years; blood pressure higher than 180/110 mm Hg at diagnosis; significant targetorgan injury at diagnosis; hemorrhages and exudates on fundus examination; renal insufficiency; LVH; poor response to appropriate three-drug therapy; and accelerated or malignant hypertension. The clinician should inquire about a family history of hypertension, premature CV disease, and disorders that would increase the possibility of secondary hypertension. The examination should include at least two standardized measurements of blood pressure with the patient in the seated position. Initially, blood pressure should also be measured in the opposite arm (to identify arterial narrowing, which can cause an inaccurately low reading in one arm) and in the standing position, especially in diabetic patients and older patients (to identify orthostatic declines). Retinal examination should be performed, primarily to identify retinal changes of diabetes or severe hypertension (i.e., hemorrhages, exudates, or papilledema). (Answer: A—A retinal examination should now be performed on this patient)

16. A 51-year-old white man recently relocated to the area and presents to your office as a new patient. He denies having any history of medical problems. He made very infrequent visits to a primary care provider where he previously lived. He is on no medicines and denies having any significant family medical history. He is a current smoker with a 40 pack-year smoking history. His blood pressure is 170/95 mm Hg, and a fourth heart sound is present. His physical examination is otherwise unremarkable.

Which of the following statements regarding treatment of this patient’s hypertension is true?
A. The most appropriate initial medical therapy for this patient is an alpha blocker
B. The most appropriate initial medical therapy for this patient is a thiazide diuretic
C. The most appropriate initial medical therapy for this patient is a thiazide diuretic in combination with another antihypertensive agent that works via a different blood pressure regulatory pathway
D. To have this patient stop smoking cigarettes would have little or no effect on the control of his hypertension

Key Concept/Objective: To understand the importance of appropriate drug therapy and lifestyle modifications in the treatment of hypertension

The Seventh Report of the Joint National Committee on Prevention, Detection, Evaluation, and Treatment of High Blood Pressure (JNC VII) recommends thiazide diuretics as initial drugs of choice for most patients with hypertension; this recommendation is based on the totality of data from randomized trials, including the Antihypertensive and Lipid Lowering Treatment to Prevent Heart Attack Trial (ALLHAT). The JNC VII report suggests initiation of therapy with two drugs rather than a single agent if the systolic blood pressure is higher than 20 mm Hg above the treatment goal or if the diastolic blood pressure is higher than 10 mm Hg above the goal. Generally, a two-drug regimen should include a diuretic appropriate for the level of renal function. The alpha-blocker arm of the ALLHAT trial was terminated early because of an almost twofold increase in the risk of heart failure, compared with the diuretic group. On the basis of these results, alpha blockers are no longer considered an appropriate initial therapy for hypertension. Tobacco use should be discouraged because, in addition to being a powerful CV risk factor, each cigarette smoked elevates blood pressure for 15 to 30 minutes, and multiple cigarettes can raise blood pressure for most of the day. (Answer: C—The most appropriate initial medical therapy for this patient is a thiazide diuretic in combination with another antihypertensive agent that works via a different blood pressure regulatory pathway)

17. A 40-year-old white woman whom you have been treating for resistant hypertension presents for routine follow-up. Laboratory results of screening tests were normal. There is electrocardiographic evidence of LVH. The patient’s lipid profile is normal. The patient has been receiving a thiazide diuretic, a calcium channel blocker, and an ACE inhibitor at near maximal doses for several weeks. Her blood pressure today remains very elevated, at 175/100 mm Hg. She experienced a minimal response to titration of her antihypertensive medicines. With the exception of elevated blood pressure and a fourth heart sound, the patient’s physical examinations have been consistently normal. Today, the physical examination is unchanged, except that you notice a soft bruit in the left upper quadrant.

Which of the following statements regarding renovascular hypertension is true?
A. Renovascular hypertension is an exceptionally rare cause of hypertension in patients with treatment-resistant hypertension
B. Stenosing lesions of the renal circulation cause hypertension through ischemia-mediated activation of the renin-angiotensin-aldosterone system
C. Fibromuscular disease is an uncommon cause of renovascular hypertension in patients of this age
D. Atheromatous disease and fibromuscular disease are equally frequent causes of renovascular hypertension

Key Concept/Objective: To know the characteristics of renovascular hypertension that is mediated by atheromatous lesions and fibromuscular disease

Renovascular hypertension is the most common form of potentially curable secondary hypertension. It probably occurs in 1% to 2% of the overall hypertensive population. The prevalence may be as high as 10% in patients with resistant hypertension, and it may be even higher in patients with accelerated or malignant hypertension. Stenosing lesions of the renal circulation cause hypertension through ischemia-mediated stimulation of the renin-angiotensin-aldosterone axis. Fibromuscular disease is the most common cause of renovascular hypertension in younger patients, especially women between 15 and 50 years of age; it accounts for approximately 10% of cases of renovascular hypertension. Atheromatous disease is the most common cause of renovascular hypertension in middle-aged and older patients and accounts for approximately 90% of cases of renovascular hypertension. The prevalence of atheromatous renal artery disease increases with age and is common in older hypertensive patients, especially in those with diabetes or with atherosclerosis in other vascular beds. Most patients with atheromatous renal vascular disease and hypertension have essential hypertension. (Answer: B—Stenosing lesions of the renal circulation cause hypertension through ischemia-mediated activation of the renin-angiotensin-aldosterone system)

18. A 55-year-old man presents to establish primary care. His medical history is significant only for 40 packyears of smoking. He drinks four beers a night. He is minimally physically active. On physical examination, the patient’s blood pressure is 158/98 mm Hg, and he is moderately obese (body mass index, 27); the rest of his examination is normal. His laboratory examination, including a chem 7, CBC, TSH, and urinalysis, is normal, as is his electrocardiogram. Repeated blood pressure measurements over the next month are similar to the values first obtained.

With respect to this patient’s blood pressure, what therapeutic option should be offered to this patient now?
A. No treatment
B. Continued monitoring for 6 months
C. Lifestyle modifications, including decreased alcohol consumption, weight loss, smoking cessation, and moderate exercise for 6 months
D. Pharmacologic therapy

Key Concept/Objective: To know the appropriate intervention, based on the patient’s hypertensive stage and risk group

This patient has stage 1 (systolic, 140 to 159 mm Hg; diastolic, 90 to 99 mm Hg), or mild, hypertension. Given his smoking history, he has greater than or equal to 1 risk factor for CV disease, which puts him in risk group 2. On the basis of the JNC VI (Joint National Committee on Prevention, Detection, and Treatment of High Blood Presssure) recommendations, it is appropriate to try lifestyle modifications (weight loss, dietary modification such as adherence to the DASH [Dietary Approaches to Stop Hypertension] diet, and moderate exercise) for 6 months before starting medications. (Answer: C—Lifestyle modifications, including decreased alcohol consumption, weight loss, smoking cessation, and moderate exercise for 6 months)

19. The patient in Question 18 adhered to your recommendations, but his blood pressure remains elevated to the same degree. He is interested in controlling his blood pressure but is worried about the cost of medications.

What should be the first-line pharmacologic therapy for this patient

A. Lisinopril, 5 mg/day
B. Extended-release diltiazem, 120 mg/day
C. Amlodipine, 5 mg/day
D. Hydrochlorothiazide, 25 mg/day

Key Concept/Objective: To know the recommended first-line medications for treatment of hypertension

Thiazide diuretics for the treatment of high blood pressure have been shown most consistently to have the best outcomes with respect to stroke and CV disease, mortality, and patient compliance. Given that the benefits of treating high blood pressure accrue only over the long term, the last of these attributes is especially important. Also, hydrochlorothiazide is by far the least expensive of all of the medications listed. (Answer: D— Hydrochlorothiazide, 25 mg/day)

20. Three months after starting therapy, the patient in Question 18 returns for follow-up. His blood pressure is 145/92 mm Hg, and blood pressure values that he has obtained outside the clinic are similar. He says that he has been taking hydrochlorothiazide as directed and has noted no unpleasant side effects. He is doing his best to adhere to the lifestyle modifications that you recommended.

What is the best step to take next in the management of this patient?
A. Continue present management
B. Add atenolol, 25 mg/day
C. Double the dose of hydrochlorothiazide to 50 mg/day
D. Add amlodipine, 5 mg/day

Key Concept/Objective: To understand the goals of antihypertensive therapy and to be able to select an appropriate second medication to achieve those goals

The goal for the treatment of hypertension is a blood pressure lower than 140/90 for most people (although this number is arbitrary, and some experts recommend still lower targets). Given that your patient is compliant with his current therapy and has done as much as he can to achieve lifestyle modification, it is appropriate to add a second agent. Atenolol is the best choice because of its low cost and proven mortality benefit. Doses of hydrochlorothiazide higher than 25 mg/day will not improve blood pressure control, and higher doses of hydrochlorothiazide have been associated with increased mortality. Amlodipine is a reasonable choice, but it is expensive, and there are no data to suggest that the calcium channel blockers improve mortality. (Answer: B—Add atenolol, 25 mg/day)

21. A 72-year-old woman comes to see you to establish care, after her previous physician retired. Her medical history is significant for diet-controlled diabetes and a myocardial infarction. She is taking aspirin, simvastatin, and amlodipine. On examination, her blood pressure is 170/95 mm Hg. She has an S4 gallop and 1+ pretibial edema.

What should be the target blood pressure in the long term for this patient?
A. < 150/90 mm Hg
B. < 140/90 mm Hg
C. < 130/85 mm Hg
D. < 120/70 mm Hg

Key Concept/Objective: To know the goal for blood pressure control for patients with evidence of target-organ disease and diabetes

This patient has stage 2 (moderate) hypertension and is in risk group C because of her history of myocardial infarction and diabetes (either one alone would suffice to put her in this risk group). As such, her target blood pressure is less than 130/85 mm Hg. Note that there is some evidence that lowering patients’ diastolic blood pressure to less than 70 mm Hg is associated with increased mortality. (Answer: C— < 130/85 mm Hg)

22. Your nurse alerts you that a patient in your clinic has severely elevated blood pressure. The patient is a
45-year-old man without other significant medical history. The patient’s blood pressure is 220/125 mm Hg; blood pressure measurements are essentially the same in both arms. The patient says that he is feeling fine. He has had no symptoms of flushing, sweating, or headache, nor has he had visual changes, focal weakness, numbness, chest pain, dyspnea, or decreased urine output. On examination, neurologic status is normal. An S4 gallop and trace pretibial edema are noted. The lungs are clear to auscultation. An ECG shows sinus rhythm with LVH. There is no evidence of ischemia or infarction.

How should you manage this patient?
A. Administer sublingual nifedipine, 10 mg
B. Administer atenolol, 50 mg, and follow up in 24 hours
C. Prescribe atenolol, 50 mg, and follow up in 1 week
D. Admit him to the intensive care unit for cardiac and blood pressure monitoring and intravenous nitroprusside therapy

Key Concept/Objective: To be able to recognize and treat severe high blood pressure

This patient has severe hypertension but no evidence of acute target-organ (brain, heart, aorta, kidney) damage or secondary causes of hypertension. In all likelihood, his hypertension has been long-standing, as suggested by his S4 gallop and LVH. Thus, he is best managed by a long-acting antihypertensive and very close follow-up to ensure that the hypertension has not worsened. He should also be educated about the symptoms of targetorgan damage and told to seek emergency care should they occur. Sublingual short-acting nifedipine has been associated with stroke, myocardial infarction, and death, and its use is very strongly discouraged. Because the patient does not have evidence of acute targetorgan damage, hospital admission is not necessary. (Answer: B—Administer atenolol, 50 mg, and follow up in 24 hours)

For more information, see Schwartz GL, Sheps SG: 1 Cardiovascular Medicine: III Hyptertension. ACP Medicine Online (www.acpmedicine.com). Dale DC, Federman DD, Eds. WebMD Inc., New York, January 2004

Atrial Fibrillation

23. A previously healthy 47-year-old woman presents with a complaint of palpitations of sudden onset. Her symptoms have been present for 12 hours. Her evaluation includes an electrocardiogram, which reveals atrial fibrillation (AF) with a ventricular response of 135 beats/min. Her cardiac examination is unremarkable except for an irregularly irregular pulse. She has no other medical problems and takes no medications. A transthoracic echocardiogram (TTE) is normal.

Which of the following is the most likely classification of this patient’s AF according to guidelines from the American College of Cardiology (ACC), the American Heart Association (AHA), and the European Society of Cardiology (ESC)?
A. Permanent
B. Persistent
C. Recurrent
D. Lone

Key Concept/Objective: To understand the appropriate classification of AF

The ACC/AHA/ESC guidelines include the following categories: recurrent—more than one episode of AF has occurred; lone—AF occurring in a patient younger than 60 years who has no clinical or echocardiographic evidence of cardiopulmonary disease; valvular—AF occurring in a patient with evidence or history of rheumatic mitral valve disease or prosthetic heart valves is defined as valvular; paroxysmal—AF that typically lasts 7 days or less, with spontaneous conversion to sinus rhythm; persistent—AF that typically lasts longer than 7 days or requires pharmacologic or direct current (DC) cardioversion; permanent— AF that is refractory to cardioversion or that has persisted for longer than 1 year. Paroxysmal, persistent, and permanent AF categories do not apply to episodes of AF lasting 30 seconds or less or to episodes precipitated by a reversible medical condition. Reversible conditions include acute myocardial infarction, cardiac surgery, pericarditis, myocarditis, hyperthyroidism, pulmonary embolism, and acute pulmonary disease. (Answer: D—Lone)

24. A 75-year-old woman with a history of symptomatic, recurrent, persistent nonvalvular AF comes to your office. She has been told that there are several options for the treatment of her AF.

Which of the following is true regarding establishment and maintenance of normal sinus rhythm, as compared with pharmacologic rate control?
A. Establishment and maintenance of sinus rhythm provides no survival advantage
B. Establishment and maintenance of sinus rhythm reduces thromboembolic risk
C. Establishment and maintenance of sinus rhythm improves the degree of symptomatic impairment
D. Conversion to normal sinus rhythm is rarely needed for patients with unstable angina, acute myocardial infarction, heart failure, or pulmonary edema

Key Concept/Objective: To understand that establishment and maintenance of sinus rhythm is not superior to ventricular rate control in patients with AF

Several trials compared restoration of sinus rhythm with control of ventricular rate in patients with AF. Evaluated outcomes included overall mortality, stroke, symptoms, and quality of life. Contrary to the expectations of many experts, maintenance of sinus rhythm provided no survival advantage and possibly a higher mortality when compared with ventricular rate control. Maintenance of sinus rhythm frequently requires the use of antiarrhythmic medications that may precipitate ventricular arrhythmias, bradycardia, and depression of left ventricular function. It was further theorized that maintenance of sinus rhythm would reduce rates of thromboembolism and the need for anticoagulation; however, trial results demonstrated no significant reduction in thromboembolic risk. Peak exercise capacity may improve with maintenance of sinus rhythm, but the two treatment strategies result in a similar degree of perceived symptomatic impairment. Nevertheless, ventricular rate control frequently is not feasible because of the complications that patients experience while in AF. AF often cannot be tolerated by patients with unstable angina, acute myocardial infarction, heart failure, or pulmonary edema. (Answer: A— Establishment and maintenance of sinus rhythm provides no survival advantage)

25. An 81-year-old man with a history of symptomatic permanent AF presents to your office to discuss options for reestablishing sinus rhythm. He hopes to decrease his symptoms of dyspnea. In addition to AF, the patient has congestive heart failure and echocardiographically documented significant mitral regurgitation.

Which of the following is NOT a risk factor for cardioversion failure in this patient?
A. Duration of AF of longer than 1 year
B. Older age
C. Left atrial enlargement
D. Normal-sized heart

Key Concept/Objective: To know the risk factors associated with failed synchronized DC cardioversion

Although success rates are high with DC cardioversion, a number of risk factors for cardioversion failure have been identified. These include longer duration of AF (notably, longer than 1 year), older age, left atrial enlargement, cardiomegaly, rheumatic heart disease, and transthoracic impedance. Pretreatment with amiodarone, ibutilide, sotalol, flecainide, propafenone, disopyramide, and quinidine have been shown to increase DC cardioversion success rates. (Answer: D—Normal-sized heart)

For more information, see Aizer A, Fuster F: 1 Cardiovascular Medicine: IV Atrial Fibrillation. ACP Medicine Online (www.acpmedicine.com). Dale DC, Federman DD, Eds. WebMD Inc., New York, December 2003

Supraventricular Tachycardia

26. A 29-year-old white woman presents to the emergency department with the complaint that her heart is “racing away.” The patient reports that this symptom began 1 hour ago and that it is associated with mild shortness of breath. She also reports having had similar episodes in her life, but she says they never lasted this long and that they usually abated with a simple cough. On examination, the patient’s pulse is regular at 175 beats/min. Her lungs are clear, and she is in mild distress. Electrocardiography reveals atrioventricular nodal reentry tachycardia (AVNRT).

Which of the following statements regarding AVNRT is false?
A. Most cases of AVNRT begin with a premature ventricular contraction (PVC)
B. Acute therapy includes carotid sinus massage and I.V. adenosine
C. Long-term therapy includes beta blockers, calcium channel blockers, and digoxin
D. Catheter ablation for AVNRT is clearly the procedure of choice for patients in whom drug therapy fails

Key Concept/Objective: To understand the pathogenesis of and therapy for AVNRT

The normal AV node has a single transmission pathway. In two to three persons per 1,000 population, however, the AV node has both a normal (fast) pathway and a second (slow) pathway. In such persons, the sinus impulse is ordinarily transmitted over the fast pathway to the ventricle, and slow-pathway conduction is preempted. However, if an atrial premature complex (APC) occurs at a critical point in the conduction cycle, the impulse can become blocked in the fast pathway, thus allowing for anterograde (forward) conduction over the slow pathway and retrograde (backward) conduction over the fast pathway. This may produce a single echo beat (a beat that returns to the chamber of origin), or it may stabilize into a circus-movement tachycardia. The diagnosis of AVNRT can usually be made by careful analysis of the 12-lead ECG. Because retrograde conduction over the AV node is occurring more or less simultaneously with anterograde conduction to the ventricles, the P wave is either buried within the QRS complex or inscribed just after the QRS. AVNRT may respond to carotid sinus massage but is highly responsive to intravenous adenosine, beta blockers, or calcium channel blockers. If carotid massage fails to convert supraventricular tachycardia, the drug of choice is intravenous adenosine, which is effective in 95% of cases. A wide variety of drugs have proved effective for controlling episodes of AVNRT, including beta blockers, calcium channel blockers, and digoxin. Long-term drug therapy is associated with frequent recurrences and adverse effects, however. Catheter ablation for AVNRT has proved so safe and effective that it is clearly the procedure of choice for patients in whom drug therapy fails. Moreover, it can be offered to those patients with milder symptoms who prefer to avoid long-term drug therapy. (Answer: A—Most cases of AVNRT begin with a premature ventricular contraction [PVC])

27. A 19-year-old man presents to the emergency department complaining of dyspnea and palpitations of acute onset. He has been short of breath for 2 hours now but denies having any chest pain. He has never had these symptoms before, and he denies having any cardiac disorders in the past. He is taking no medicines and has no significant family history of sudden cardiac death or arrhythmias. On examination, the patient is tachycardic but the heartbeat is regular. The patient’s lungs are clear. His blood pressure is
110/72 mm Hg, and he is afebrile. ECG reveals a narrow complex tachycardia with a retrograde P wave noted in the ST segment. You diagnose the patient as having atrioventricular reentry tachycardia (AVRT).

Which of the following statements regarding AVRT is true?

A. AVRT usually improves during pregnancy
B. Because of the location of the reentrant pathway, catheter ablation is contraindicated
C. During an episode of atrial fibrillation in a patient with WolffParkinson-White (WPW) syndrome, the drug of choice for initial management is a calcium channel blocker
D. Patients with WPW syndrome are at risk for sudden cardiac death from ventricular fibrillation

Key Concept/Objective: To understand the pathogenesis and treatment of WPW syndrome

The most prominent manifestation of accessory atrioventricular pathways is the WPW syndrome. In this syndrome, the accessory pathway can be located at various regions around the tricuspid and the mitral atrioventricular rings, but it is most commonly sited at the left free wall of the mitral annulus. The basic mechanism of tachycardia in AVRT is similar to that of AVNRT. Electrical impulses can travel down both the AV node and the accessory pathway to activate the ventricles, with ventricular activation occurring earlier at sites near the accessory pathway than at sites activated normally (i.e., ventricular preexcitation). The most feared arrhythmia in the WPW syndrome involves atrial fibrillation with dominant conduction over an accessory pathway that has rapid conduction properties. These patients may experience extraordinarily rapid ventricular rates and are at risk for sudden cardiac death from ventricular fibrillation. Symptomatic tachyarrhythmias associated with the WPW syndrome generally begin in the teenage years or during early adulthood. Pregnancy may produce an initial attack in some women. Pregnancy can also be associated with an increasing frequency of attacks and more symptomatic episodes. Ventricular preexcitation may be evident on a baseline ECG as fusion complexes (WPW pattern). The WPW pattern comprises a short PR interval and an earlier-than-normal deflection on the QRS complex (delta wave). The ECG during AVRT will usually show a narrow complex with the retrograde P wave falling in the ST segment, because atrial activation occurs well after ventricular depolarization. The acute management of AVRT is similar to that for AVNRT: adenosine is the drug of choice, but calcium channel blockers or beta blockers are also effective. Long-term therapy for AVRT may be directed at interfering with conduction either through the AV node (i.e., with beta blockers or calcium channel blockers) or through the accessory pathway (i.e., with class IC or class III antiarrhythmic agents). The remarkable efficacy and safety of ablation make this mode of therapy more attractive than long-term drug therapy for symptomatic patients. Drug therapy carries the possibility of recurrent arrhythmias, including atrial fibrillation. Hence, ablation is currently recommended for all patients with symptomatic WPW. (Answer: D—Patients with WPW syndrome are at risk for sudden cardiac death from ventricular fibrillation)

For more information, see Tchou PJ, Trohman RG: 1 Cardiovascular Medicine: V Supraventricular Tachycardia. ACP Medicine Online (www.acpmedicine.com). Dale DC, Federman DD, Eds. WebMD Inc., New York, August 2004

Pacemaker Therapy

28. A 60-year-old man presents to his primary care physician for evaluation of dizziness and increased fatigue. An electrocardiogram is performed as part of his evaluation. The ECG demonstrates complete heart block, with a ventricular rate of 44 beats/min. The patient is referred for implantation of a pacemaker.

Which of the following is NOT an indication for implantation of a cardiac pacemaker?
A. Temporary pacing in the setting of acute myocardial infarction complicated by conduction abnormalities and hemodynamic instability
B. Resynchronization in the treatment of heart failure
C. Type I second-degree atrioventricular (AV) block in an asymptomatic athlete
D. Complete AV block
E. Neurocardiogenic syncope with significant bradycardia

Key Concept/Objective: To know the various indications for cardiac pacing

Conduction abnormalities are common in the setting of acute myocardial infarction. Patients with acute inferior infarction can manifest a variety of abnormalities, including sinoatrial (SA) node dysfunction, first-degree AV block, type I second-degree block, and third-degree block at the level of the AV node. It is uncommon for any of these conduction disturbances to persist after the acute phase of the infarction. These patients often require temporary pacing if they manifest hemodynamic instability. Cardiac resynchronization therapy is an exciting new development in the treatment of heart failure. Complete AV block with bradycardia and the presence of symptoms is an indication for permanent cardiac pacing. Classic neurocardiogenic syncope involves sinus tachycardia followed by bradycardia, vasodilatation, and syncope. Some patients have primarily a vasodepressive (vasodilatation) syndrome, whereas others have a syndrome with a significant cardioinhibitory component (bradycardia). In the setting of bradycardia, cardiac pacemaker implantation is necessary. It is not uncommon for trained athletes to have type I second-degree AV block and be asymptomatic. Pacemaker therapy is not indicated. (Answer: C—Type I second-degree atrioventricular (AV) block in an asymptomatic athlete)

29. A 67-year-old female patient of yours is admitted to the hospital. She has a permanent pacemaker and sees a cardiologist. In reviewing her chart, you note that her pacemaker program code is VVI, with a lower rate of 60 beats/min.

Which of the following statements regarding this patient’s pacemaker program code is false?
A. Both the atria and ventricles are programmed to be paced
B. The sensing lead is in the ventricle
C. When the intrinsic heart rate falls below 60 beats/min, pacing will occur
D. After a paced beat, the pacemaker clock resets and senses the next ventricular contraction
E. VVI is also referred to as ventricular demand pacing or ventricular inhibited pacing

Key Concept/Objective: To understand the three-letter code for describing the basic functions of cardiac pacemakers

The three basic functions of a pacemaker—pacing, sensing, and action—are determined by basic pacemaker programming. In 1974, the American Heart Association and the American College of Cardiology proposed a three-letter code for describing the basic functions of pacemakers. Under the guidance of the North American Society of Pacing and Electrophysiology (NASPE) and the British Pacing and Electrophysiology Group (BPEG), this code evolved into the five-position code currently in use. The first position denotes the chamber or chambers paced; the second denotes the chamber or chambers sensed; the third denotes the action or actions performed; the fourth denotes rate response; and the fifth denotes multiple site pacing. The simplest mode of pacing is VVI, otherwise known as ventricular demand pacing or ventricular inhibited pacing. The most commonly used mode in dual-chamber pacing is DDD. The most basic timing cycle is the lower rate, which reflects how long the pacemaker will wait after a paced or sensed beat before initiating pacing. If the pacemaker is set to VVI mode at a lower rate of 60 beats/min, then as long the interval between intrinsic beats is less then 1,000 msec, the pacemaker will reset the lower rate clock with each sensed QRS complex, and pacing will not occur. If, however, the intrinsic heart rate falls below 60 beats/min, the pacemaker’s lower rate clock will time out before an intrinsic beat is sensed, and pacing will occur. After a paced beat, the lowerrate clock is reset and the cycle repeats. (Answer: A—Both the atria and ventricles are programmed to be paced)

30. A 56-year-old woman is admitted for implantation of a permanent pacemaker for management of sick sinus syndrome. The procedure is successful.

Which of the following statements regarding further care of this patient is true?
A. It is standard practice to discharge the patient the day of the procedure if no obvious complications occurred
B. There is no need for telemetric monitoring if admitted
C. A chest radiograph is routinely performed to verify lead position and to evaluate for pneumothorax
D. The rate of adverse events associated with pacemaker implantation is 1%
E. Once the pacemaker has been installed, there is no need for interrogating the device

Key Concept/Objective: To understand the immediate complications associated with pacemaker implantation and appropriate postimplantation care

Overall, transvenous pacemaker implantation is both safe and well tolerated. The risk of major adverse events (e.g., death, myocardial infarction, stroke, and the need for emergency thoracotomy) is approximately 0.1%. Other complications sometimes encountered include pneuomothorax, vascular injury, cardiac perforation, tamponade, local bleeding, pocket hematoma, infection, and venous thrombosis. At most institutions, it is standard practice to admit patients for overnight observation after routine pacemaker implantation. Patients are monitored via continuous telemetry. We routinely obtain a portable chest x-ray and a 12-lead ECG immediately after implantation. The day after the procedure, the pacemaker is interrogated and the final settings confirmed. Posteroanterior and lateral chest x-rays are obtained both to verify the positioning of the leads and to rule out the possibility of a slowly accumulating pneumothorax. (Answer: C—A chest radiograph is routinely performed to verify lead position and to evaluate for pneumothorax)

31. A 76-year-old man with a permanent pacemaker is admitted to the hospital with a diagnosis of pneumonia. The patient unfortunately develops respiratory failure and is intubated. A central venous line is placed for administration of antibiotics and pressors. He improves clinically but develops fever. Blood cultures are positive for Staphylococcus aureus. Appropriate antimicrobial therapy is instituted, and the central line is removed. The patient remains febrile with persistently positive cultures.

Which of the following statements regarding pacemaker infection is true?
A. The most common organism causing pacemaker infection is S. aureus
B. Pacemaker infection is easily treated with appropriate antimicrobial therapy
C. Patients with S. aureus bacteremia are not at significant risk for secondary pacemaker infection
D. Infected pacemaker leads can simply be exchanged, as opposed to removing the entire unit
E. Patients with infected pacemaker hardware need to be sent to a referral center with experience in removing these devices

Key Concept/Objective: To understand that pacemaker infection requires special expertise and that patients should be referred to special centers with experience in device removal and pacemaker infection therapy

Bacterial infections can affect any part of the pacemaker system, and the consequences can be devastating. The most common pathogens are staphylococci, especially S. epidermidis. Once a pacemaker infection is established, it is difficult to eradicate with antibiotics; thus, infected pacemaker systems usually must be removed in their entirety. Patients with pacemakers in place who acquire S. aureus bacteremia are at significant risk for a secondary device infection. If an infection of an implanted cardiac device is suspected, prompt referral to an experienced center is critical. (Answer: E—Patients with infected pacemaker hardware need to be sent to a referral center with experience in removing these devices)

For more information, see Lowy J, Freedman RA: 1 Cardiovasculaar Medicine: VII Pacemaker Therapy. ACP Medicine Online (www.acpmedicine.com). Dale DC, Federman DD, Eds. WebMD Inc., New York, May 2004

Acute Myocardial Infarction

32. A 53-year-old black man presents to the emergency department with a complaint of chest pain of 2 hours’ duration. The pain woke him from sleep. It is substernal and radiates to his left shoulder. The patient has vomited twice and is diaphoretic. He has no history of coronary artery disease but has hypertension and hypercholesterolemia.

Which of the following statements regarding acute myocardial infarction (MI) is false?
A. The presence of a severe stenosis (i.e., a stenosis = 70% of the diameter of the artery), as seen on coronary angiography, correlates well with the most vulnerable sites for plaque rupture and occlusion of the coronary artery
B. Within 10 minutes of arrival at the emergency department, a patient with symptoms suggestive of an MI should be evaluated; subsequently, the patient should be evaluated with a 12-lead electrocardiogram, and oxygen and aspirin should be administered
C. Morphine sulfate is acceptable for pain control in a patient with an acute MI
D. In the second International Study of Infarct Survival (ISIS-2) trial, aspirin was found to be nearly as effective as streptokinase in reducing 30-day mortality

Key Concept/Objective: To understand the basic pathophysiology and the initial treatment of acute MI

Although to produce anginal symptoms, a stenosis of a coronary artery must be severe (i.e.,
= 70% of the diameter of the artery), such stenoses tend to have dense fibrotic caps and are less prone to rupture than mild to moderate stenoses, which are generally more lipid laden. Studies of patients in whom angiography was performed before and after an MI revealed that in most cases, acute coronary occlusion occurred at sites in the coronary circulation with stenoses of less than 70%, as demonstrated on the preinfarction angiogram. A patient with symptoms suggestive of MI should be evaluated within 10 minutes after arrival in the emergency department. Early steps should include the assessment of hemodynamic stability by measurement of the patient’s heart rate and blood pressure; the performance of a 12-lead ECG; and the administration of oxygen by nasal prongs, of I.V. analgesia (most commonly morphine sulfate), of oral aspirin, and of sublingual nitroglycerin if the blood pressure is greater than 90 mm Hg. (Answer: A—The presence of a severe stenosis [i.e., a stenosis = 70% of the diameter of the artery], as seen on coronary angiography, correlates well with the most vulnerable sites for plaque rupture and occlusion of the coronary artery)

33. A 62-year-old white woman with a history of coronary artery disease presents to the emergency department with substernal, squeezing chest tightness of 2 hours’ duration. The pain is identical to the pain she experienced with her first MI. On physical examination, the patient’s heart rate is found to be 105 beats/min; a tachycardic regular rhythm without gallop is noted. The patient’s lung fields are clear. A chest radiograph is normal, but ECG reveals ST segment elevation in leads I, aVL, V5, and V6.

Which of the following statements regarding the management of this patient is true?
A. Thrombolytic therapy has been studied in patients with ECG findings other than ST segment elevation or bundle branch block and has been found to be superior to conventional therapy
B. Current recommendations are that the time between a patient’s presentation to the emergency department and the administration of thrombolytic therapy not exceed 2 hours
C. Coronary angiography is recommended in all patients after thrombolytic therapy has been administered, once they become hemodynamically stable
D. Streptokinase therapy is contraindicated in patients who have recently received a dose of streptokinase because of antibodies that form against the drug

Key Concept/Objective: To understand the basic principles of thrombolytic therapy

The time between a patient’s presentation to the emergency department and the administration of thrombolytic therapy should not exceed 60 minutes. Front-loaded tissue plasminogen activator (t-PA) has been found to be superior to the other thrombolytic regimens. However, some physicians prefer the less expensive streptokinase therapy, particularly for patients at low risk of dying (e.g., those with uncomplicated inferior infarctions) and the elderly, who are more likely to have hemorrhagic complications with t-PA than with streptokinase. Streptokinase is contraindicated in patients who have recently received a dose of streptokinase because of antibodies that form against the drug; these antibodies limit the efficacy of repeat doses and increase the risk of allergic reactions. Thrombolytic therapy has been studied in patients with ECG findings other than ST segment elevation or bundle branch block and has been found to be either of no use or deleterious. Patients treated with thrombolytic therapy in whom complications do not occur are at low risk for reinfarction and death after discharge, and routine performance of coronary angiography and coronary angioplasty does not reduce the occurrence of these adverse events. Coronary angiography is recommended only for patients with hemodynamic instability or for patients in whom spontaneous or exercise-induced ischemia occurs. (Answer: D—Streptokinase therapy is contraindicated in patients who have recently received a dose of streptokinase because of antibodies that form against the drug)

34. A 49-year-old white man who presented to the emergency department with an ST segment elevation MI was given thrombolytics, oxygen, and aspirin. He is now free of chest pain and will be admitted to the coronary care unit for further monitoring.

Which of the following statements regarding adjuvant medical therapy for acute MI is false?
A. Early administration of beta blockers reduces the mortality and the reinfarction rate
B. Unless contraindicated, angiotensin-converting enzyme (ACE) inhibitors are indicated in patients with significant ventricular dysfunction after acute MI
C. When given within 6 hours after presentation to the hospital, I.V. nitroglycerin reduces mortality in patients with MI
D. Prophylactic therapy with lidocaine does not reduce and may actually increase mortality because of an increase in the occurrence of fatal bradyarrhythmia and asystole

Key Concept/Objective: To understand the adjuvant medical therapies available for patients with acute MI after reperfusion therapy has been administered

Early administration of beta blockers may reduce infarct size by reducing heart rate, blood pressure, and myocardial contractility. It is recommended that all patients with acute MI without contraindications receive I.V. beta blockers as early as possible, whether or not they receive reperfusion therapy. Several large, randomized, controlled clinical trials evaluated the use of ACE inhibitors early after acute MI; all but one trial revealed a significant reduction in mortality. To determine whether nitroglycerin therapy is beneficial in patients treated with reperfusion, 58,050 patients with acute MI in the ISIS-4 trial were randomized to receive either oral controlled-release mononitrate therapy or placebo; thrombolytic therapy was administered to patients in both groups. The results of this study revealed no benefit from the routine administration of oral nitrate therapy in this setting. Previously, routine prophylactic antiarrhythmic therapy with I.V. lidocaine was recommended for all patients in the early stages of acute MI. However, studies have revealed that prophylactic therapy with lidocaine does not reduce and may actually increase mortality because of an increase in the occurrence of fatal bradyarrhythmia and asystole. (Answer: C—When given within 6 hours after presentation to the hospital, I.V. nitroglycerin reduces mortality in patients with MI)

35. A 49-year-old white woman was admitted last night with an acute ST segment elevation MI. She underwent left heart catheterization with restoration of blood flow to her left circumflex artery and is currently in the CCU. She has received anticoagulation therapy and has been started on an ACE inhibitor, aspirin, and a beta blocker.

Which of the following statements regarding possible complications of acute MI is true?
A. The Atrial Fibrillation Follow-up Investigation of Rhythm Management (AFFIRM) trial showed that rhythm-control strategies provided a significant survival advantage when compared with rate-control strategies
B. Beta blockers may reduce the early occurrence of ventricular fibrillation
C. Severe mitral regurgitation is 10 times more likely to occur with anterior MI than with inferior MI
D. When patients have right ventricular infarction, the left ventricle is almost always spared of any damage

Key Concept/Objective: To know the complications associated with acute MI

Although lidocaine has been shown to reduce the occurrence of primary ventricular fibrillation, mortality in patients receiving lidocaine was increased because of an increase in fatal bradycardia and asystole, and prophylactic lidocaine is no longer recommended if defibrillation can rapidly be performed. Beta blockers may reduce the early occurrence of ventricular fibrillation and should be administered to patients who have no contraindications. The treatment of atrial fibrillation in acute MI should be similar to the treatment of atrial fibrillation in other settings. If atrial fibrillation recurs, antiarrhythmic agents may be used, although their impact on clinical outcomes is unproven. Mild mitral regurgitation is common in acute MI and is present in nearly 50% of patients. The posterior papillary muscle receives blood only from the dominant coronary artery (the right coronary artery in nearly 90% of patients); thrombotic occlusion of this artery may cause rupture of the posterior papillary muscle, resulting in severe mitral regurgitation. Although nearly all patients with right ventricular infarction suffer both right and left ventricular infarction, the characteristic hemodynamic findings of right ventricular infarction generally dominate the clinical course and must be the main focus of therapy. (Answer: B—Beta blockers may reduce the early occurrence of ventricular fibrillation)

36. A 55-year-old white man presented to the emergency department with crushing substernal chest pain of
1 hour’s duration that radiated to his left arm; associated with this pain were dyspnea, diaphoresis, nausea, and vomiting. The patient’s cardiac risk factors include hypertension of 10 years’ duration, current tobacco use, and a strong family history of CAD. Examination findings were as follows: BP, 158/87 mm Hg; pulse, 105 beats/min; and lung crackles at bases. ECG showed an ST segment elevation of 3 mm in leads V2 through V5 with reciprocal ST segment depression in leads II, III, and aVF. Laboratory results showed normal CK and troponin I levels and an LDL level of 120. After administration of O2, aspirin, nitrates, and morphine, the chest pain subsided, but the ECG still shows an ST segment elevation in leads V2 through V5.

Which of the following would you consider for initial treatment of this patient?
A. Reperfusion therapy
B. Statin therapy
C. ACE inhibitor
D. Glycoprotein IIb-IIIa inhibitor

Key Concept/Objective: To understand the indications and contraindications for reperfusion therapy

This patient was initially treated for acute MI, and he now needs to receive reperfusion therapy as rapidly as possible to restore normal antegrade blood flow in the occluded artery. Reperfusion may be accomplished by thrombolytic therapy or by percutaneous transluminal coronary angioplasty (PTCA). In prospective trials, thrombolytic therapy has been shown to reduce mortality by 29% in patients with ST segment elevation who have been treated within 6 hours of the onset of chest pain. In prospective, randomized clinical trials comparing thrombolytic therapy with direct coronary angioplasty, direct coronary angioplasty was associated with lower morbidity and mortality than thrombolytic therapy. PTCA, therefore, is the preferred choice in facilities that have surgical backup where it can be performed quickly and that have a high angiographic success rate. Reperfusion therapy is contraindicated in patients who present with conditions that predispose them to significant bleeding. Thrombolytic therapy is contraindicated in patients who have ECG abnormalities other than ST segment elevation or bundle branch block. A complete and rapid assessment needs to be conducted to ensure that the right course of action is taken. The potential benefits of thrombolytic therapy may outweigh the risks of reperfusion therapy. (Answer: A— Reperfusion therapy)

37. The patient described in Question 36 has now been treated with thrombolytic therapy with t-PA.

Adding which of the following at or before the time of administration of thrombolytic therapy would likely improve this patient’s outcome?
A. Statin therapy
B. I.V. nitroglycerin
C. Glycoprotein IIb-IIIa inhibitor
D. I.V. beta-blocker therapy

Key Concept/Objective: To understand the value of beta blockers early in the treatment of MI

A number of studies of beta-blocker therapy in patients with acute MI have documented improved patient outcomes, including reductions in hospital stay and improved longterm mortality. Early administration of beta blockers may reduce infarct size by reducing heart rate, blood pressure, and myocardial contractility, all of which diminish myocardial oxygen demand. Use of I.V. beta-blocker therapy as soon as possible is recommended for all patients with acute MI in whom beta-blocker therapy is not contraindicated, whether or not patients receive reperfusion therapy. Those with the largest infarcts benefit the most from the use of beta blockers. In the absence of side effects and contraindications, beta-blocker therapy should be continued indefinitely. (Answer: D—I.V. beta-blocker therapy)

38. A 56-year-old man presents to the emergency department with complaint of chest pain of 20 minutes’ duration. The pain is severe, crushing, substernal, and without radiation. He has associated nausea and diaphoresis without vomiting. He has had no previous episodes of chest discomfort. He has not seen a doctor for over 20 years and takes no medications. He has smoked two packs of cigarettes a day for the past 35 years and has lived a sedentary lifestyle. His family history is remarkable for an MI in his father at 49 years of age. Physical examination reveals a thin man, sitting upright, breathing rapidly on 2 L of oxygen. His vital signs include the following: temperature, 98.8° F (37.1° C); pulse, 98 beats/min; respiratory rate, 22 breaths/min; blood pressure, 150/95 mm Hg. Cardiac examination reveals normal rate and rhythm without murmur, and neck veins are not elevated. Lungs are clear to auscultation. ECG shows normal sinus rhythm with occasional premature ventricular contractions and ST segment elevations of
0.2 mV in leads II, III, and aVF.

Which of the following interventions is NOT indicated for this patient at this time?
A. Morphine sulfate I.V.
B. Lidocaine
C. Aspirin
D. Metoprolol
E. Streptokinase

Key Concept/Objective: To understand the initial management of acute MI

This patient meets criteria for acute MI with a characteristic history and ECG changes. Emergent therapy should include oxygen, aspirin, analgesia, nitrates, beta blockers, and early reperfusion. The prophylactic administration of antiarrhythmic agents in the absence of significant arrhythmias does not reduce mortality and may actually increase mortality through increased incidence of bradyarrhythmias and asystole. (Answer: B— Lidocaine)

39. The patient in Question 38 receives thrombolytic therapy with streptokinase within 30 minutes of the onset of his chest pain. Soon after administration of streptokinase, the ECG changes revert to baseline. He is monitored on telemetry for 48 hours without any arrhythmias. He is able to walk around the ward without difficulty.

Further diagnostic tests during the immediate discharge period should include which of the following?
A. Cardiac catheterization
B. Nuclear imaging with pharmacologic stress
C. Stress echocardiogram
D. Low-level exercise treadmill test at 1 week post-MI
E. Symptom-limited exercise treadmill test at 1 week post-MI

Key Concept/Objective: To understand the post-MI risk-stratification strategies

The utility of cardiac catheterization post-MI has not been extensively studied, and patterns of practice vary widely. In the setting of a patient with acute MI who has successfully been treated with thrombolytic agents, the utility of post-MI catheterization has been studied in two clinical trials and has been found not to reduce the risk of reinfarction or death. The recommendation of the American College of Cardiology/American Heart Association is to use a submaximal exercise test at 5 to 7 days or a symptom-limited exercise test at 14 to 21 days. The stress echocardiogram and nuclear imaging are to be used in patients who are unable to exercise or in those whose baseline ECG has abnormalities, such as left bundle branch block or left ventricular hypertrophy with strain, that preclude interpretation of a stress test. The symptom-limited exercise treadmill test is recommended not in the immediate postdischarge period but at 3 to 6 weeks. Coronary angioplasty following positive exercise treadmill tests has been shown to improve the rates of nonfatal MI and unstable angina in the Danish Acute Myocardial Infarction (DANAMI) study. (Answer: D—Low-level exercise treadmill test at 1 week post-MI)

40. A 72-year-old woman is seen by her primary care physician. She reports 5 days of shortness of breath on exertion. Five days ago, she reported having several hours of chest discomfort, which she ascribed to indigestion, and did not seek medical attention. She currently denies any chest discomfort. Her medical history is remarkable for hypertension, type 2 diabetes mellitus, and obesity. Medications include glyburide and hydrochlorothiazide. She is a nonsmoker. Physical examination reveals a moderately obese woman, seated in a chair, breathing comfortably. Vital signs are as follows: pulse, 90 beats/min; blood pressure, 145/90 mm Hg; respiratory rate, 20 breaths/min. Examination of the heart shows a regular rate and rhythm and the presence of an S3 gallop. Neck veins cannot be assessed because of obesity. Lungs are clear to auscultation. Laboratory data are remarkable for a low-density-lipoprotein (LDL) cholesterol level of 135 mg/dl, a high-density-lipoprotein (HDL) cholesterol level of 41 mg/dl, and a triglyceride level of 220 mg/dl. ECG shows normal sinus rhythm; Q waves in V1, V2, and V3; and poor R-wave progression, which are new changes compared with an ECG of 2 years ago. Echocardiography reveals a depressed ejection fraction of 35% and no valvular pathology.

Which of the following medications is not likely to prolong survival in this patient?
A. Carvedilol
B. Simvastatin
C. Aspirin
D. Enalapril
E. Estrogen/progesterone

Key Concept/Objective: To understand the pharmacotherapy of secondary prevention after acute MI

Beta blockers have been shown to increase survival and are recommended in all post-MI patients without contraindications. Carvedilol has been studied specifically in the setting of nondecompensated congestive heart failure and has been shown to prolong survival. Lipid-lowering therapy with HMG-CoA reductase inhibitors (statins) has been shown to reduce both cardiac and all-cause mortality. Treatment guidelines in patients with known coronary heart disease establish a treatment-goal LDL cholesterol level of less than 100 mg/dl. Aspirin therapy has been shown to have significant reductions in the rate of nonfatal MI, nonfatal stroke, and vascular death. ACE inhibitors have been shown to reduce mortality post-MI and are recommended in all patients with anterior MI or with clinical heart failure in the absence of hypotension. The recently published Heart and Estrogen/ Progestin Replacement Study (HERS) showed no overall cardiovascular benefit at 5 years from estrogen/progestin replacement and an increase in early cardiovascular events compared with placebo. On the basis of this study, estrogen/progestin is not recommended as secondary prevention after acute MI. Women who are already taking hormone replacement therapy at the time of acute MI may continue to do so. (Answer: E—Estrogen/progesterone)

41. An 80-year-old man presents to the emergency department with chest pain of 30 minutes’ duration. He has a history of an inferior MI that was treated with streptokinase 6 months ago. He has hypertension, with a controlled blood pressure of 130/80 mm Hg, and has not taken his medications for several days. He says his current symptoms are similar to those he experienced previously. His medical history is remarkable for gout, hypertension, and an ischemic stroke 8 years ago. Current medications include atenolol, aspirin, and allopurinol. The patient is afebrile, his blood pressure is 170/100 mm Hg, his pulse is 90 beats/min, and his respiratory rate is 20 breaths/min. He is diaphoretic and in apparent pain. Cardiovascular examination shows a regular rate and rhythm without murmur. Lungs are clear to auscultation. ECG shows ST segment elevation greater than 0.2 mV in V2 to V6.

Which of the following features in this case would be an absolute contraindication to thrombolytic therapy?
A. Failure to meet ECG criteria
B. Age greater than 75 years
C. History of stroke
D. Elevated blood pressure
E. There are no absolute contraindications

Key Concept/Objective: To understand the indications and contraindications for thrombolytic therapy

The patient clearly meets ECG criteria for the administration of thrombolytic therapy: ST segment elevation greater than 0.1 mV in two contiguous leads. Age greater than 75 years is not a contraindication to thrombolysis. There is increased mortality in this group with and without thrombolytic therapy. In patients older than 75 years, there is an increased risk of hemorrhagic stroke, but overall mortality is reduced in such patients without contraindications. A prior history of hemorrhagic stroke is an absolute contraindication to thrombolytics, but a history of an ischemic stroke more than 1 year before presentation is a relative contraindication. Severe uncontrolled hypertension (blood pressure > 180/110 mm Hg) is a relative contraindication to thrombolytic therapy, but this patient does not have that degree of hypertension. Because this patient has had exposure to streptokinase within the past 2 years, this agent should be avoided, as antibodies against the drug will reduce its efficacy and create potential for allergic reactions. (Answer: E—There are no absolute contraindications)

42. A 77-year-old woman presents with 2 hours of chest pain, which varies in intensity from mild to severe. Her pain is described as “pressure” felt over the left chest, with radiation to the left arm. It occurred at rest and is worsened by any activity. She has nausea without vomiting. Her medical history is remarkable for an inferior MI 5 years ago, diabetes, and hypertension. Her medications include lisinopril and metformin. Physical examination reveals a moderately obese woman in apparent discomfort. Vital signs include pulse, 84 beats/min; BP, 130/80 mm Hg; respiratory rate, 16; oxygen saturation, 96% on room air. Cardiac and lung examinations are normal. Her ECG shows Q waves in III and aVF; 2 mV ST segment depression in leads V3 to V6; and 1 mV ST segment elevation in V1. She is treated initially with oxygen, sublingual nitroglycerin, aspirin, metoprolol, and morphine, and her symptoms improve. She still rates her pain as moderate, and repeat ECG is unchanged.

Which of the following would NOT be an appropriate intervention for this patient?

A. Low-molecular-weight heparin
B. Cardiac catheterization
C. Abciximab
D. Thrombolytics
E. Eptifibatide

Key Concept/Objective: To understand the management of acute coronary syndromes with a nondiagnostic ECG

This case represents an acute coronary syndrome that is not diagnostic for acute MI and does not represent chronic stable angina. Patients with nondiagnostic ECGs on presentation may have noncardiac chest pain, unstable angina, or MI. The use of low-molecularweight heparin in this setting has been studied in several trials and has been shown to have a clear benefit. Glycoprotein IIb/IIIa inhibitors (abciximab, eptifibatide, and tirofiban) block the final common pathway of platelet aggregation and have been shown to reduce the risk of death, MI, and revascularization procedures. The use of cardiac catheterization in the setting of non–ST segment acute coronary syndromes has been shown to reduce the length of stay with initial hospitalization and the need for rehospitalization. In a recent trial, an invasive strategy combined with antiplatelet therapy was shown to reduce the rate of death or MI when compared with a noninvasive strategy. Current recommendations of the American College of Cardiology/American Heart Association include catheterization and revascularization of high-risk patients (defined by prior MI, left ventricular dysfunction, widespread ECG changes, or recurrent ischemia). Thrombolytics have been studied in the setting of acute coronary syndromes without ST segment elevation or bundle branch block and have been found to be deleterious, so they are contraindicated in this setting. (Answer: D—Thrombolytics)

For more information, see Berger PB, Orford JL: 1 Cardiovascular Medicine: VIII Acute Myocardial Infarction. ACP Medicine Online (www.acpmedicine.com). Dale DC, Federman DD, Eds. WebMD Inc., New York, September 2004

Chronic Stable Angina

43. A previously healthy 52-year-old man presents with complaints of intermittent substernal chest discomfort. The pain does not radiate. The symptoms occur with exercise, and they are not relieved by rest. The patient does not have shortness of breath. The resting ECG is normal. You determine that the patient has an intermediate pretest probability of having significant coronary artery disease, and you elect to have him undergo exercise ECG testing to further evaluate his symptoms.

Which of the following findings would be most highly suggestive of significant ischemic heart disease
(IHD) on exercise ECG testing?
A. Chest discomfort before completion of the test
B. Hypertension during the test
C. An S3 heart sound during the test
D. A 0.5 mm ST segment depression during the test

Key Concept/Objective: To understand the significant positive findings during exercise ECG testing

Recently published guidelines from the American College of Cardiology/American Heart Association/American College of Physicians (ACC/AHA/ACP) recommend exercise ECG as the diagnostic test of choice for the average patient with an intermediate pretest probability of IHD and a normal resting ECG. Exercise-induced falls in blood pressure or the development of an exercise-induced S3 heart sound are strongly suggestive of ischemic left ventricular dysfunction. Specific exercise-induced ECG changes include changes = 1 mm horizontal or downward-sloping ST segment depression or elevation during or after exercise. Exercise-induced changes in lead V5 are most reliable for the diagnosis of IHD. (Answer: C—An S3 heart sound during the test)

44. A 56-year-old man with hypertension presents to your clinic for a routine health maintenance visit. He is asymptomatic and takes only hydrochlorothiazide. His blood pressure is 138/78 mm Hg. His total cholesterol level is 190 mg/dl, and his high-density liproprotein (HDL) cholesterol level is 36 mg/dl. He is a nonsmoker. He tells you he is concerned about IHD and that he has read about new methods to detect early disease, including CT imaging. He is interested in this screening test in hope of detecting any disease he may have before it becomes a problem.

How should you advise this patient with regard to electron-beam computed tomography (EBCT)?
A. You should tell him that EBCT is a safe and effective method of detecting early coronary artery disease
B. You should recommend this test because it will hopefully alleviate his concerns about IHD
C. You should recommend against this form of testing because his risk of heart disease can be equally well determined by the information already known about him
D. You should recommend against this test because even if the test is negative, he would still have a high likelihood of having significant lesions

Key Concept/Objective: To understand the limitations of new technology in the diagnosis of IHD

Although the sensitivity of EBCT for the diagnosis of significant coronary artery stenosis is high, the specificity of EBCT for significant coronary artery stenosis ranges from only
41% to 76%, yielding many false positive results. To date, no prospective, population-based studies have been performed to investigate a potential association between the calcium score derived from EBCT and the risk of future coronary events, and no studies have shown that screening for IHD with EBCT reduces mortality. Asymptomatic patients for whom EBCT results indicate a potentially high risk of cardiac events may suffer anxiety and unnecessary procedures as a result of the study. The ACC/AHA do not currently recommend EBCT and other imaging procedures, such as magnetic resonance imaging angiography, in asymptomatic patients. (Answer: C—You should recommend against this form of testing because his risk of heart disease can be equally well determined by the information already known about him)

45. A 61-year-old woman was recently admitted to the hospital with acute coronary syndrome. She was found to have coronary artery disease that is not amenable to revascularization procedures. She is hypertensive and has hyperlipidemia. She smokes approximately 1 pack of cigarettes a day. She currently has stable angina. Medical therapy and lifestyle changes are recommended for this patient.

Which of the following statements is true regarding the management of this patient?

A. Clopidogrel and ticlopidine are equally effective in reducing future cardiovascular events
B. Smoking cessation is as effective as or more effective than any current medical therapy in reducing the risk of future cardiovascular events
C. Patients with chronic stable angina should be placed on statin therapy only if their low-density lipoprotein (LDL) cholesterol level is greater than 100 mg/dl
D. It is clear that patients who walk for at least 1 hour five to seven times a week derive more benefit than patients who walk only for 30 minutes five to seven times a week

Key Concept/Objective: To understand the management of patients with IHD and chronic stable angina

A systematic review of prospective cohort studies of smokers with IHD found a striking 29% to 36% relative risk reduction in all-cause mortality for patients who were able to quit smoking. The magnitude of the risk reduction for smoking cessation was as great as or greater than that expected to result from use of aspirin, statins, beta blockers, or angiotensin-converting enzyme (ACE) inhibitors. Patients with chronic stable angina should be encouraged to include moderate aerobic activity in their daily lives. Moderate physical activity consists of walking briskly for 30 minutes or more five to seven times a week or the equivalent. There are no studies demonstrating that ticlopidine reduces cardiovascular events in outpatients with chronic stable angina. Results of the Heart Protection Study indicate that all patients with chronic stable angina should be treated with a statin, barring specific allergy. (Answer: B—Smoking cessation is as effective as or more effective than any current medical therapy in reducing the risk of future cardiovascular events)

46. A 72-year-old male patient has long-standing IHD. He has significant angina that is stable but causes him considerable distress and limits his activities of daily living. You hope to improve his anginal symptoms.

For this patient, which of the following statements regarding the management of the symptoms of chronic stable angina is true?
A. Because of bronchospasm, beta blockers are usually not well tolerated in patients with chronic obstructive pulmonary disease
B. Although both beta blockers and calcium channel blockers are effective in the treatment of angina, the combination of these two medicines offers little additional benefit
C. Patients with left ventricular systolic dysfunction should never be started on calcium channel blockers because of their negative inotropic effects
D. Nitrates should not be used as treatment for anginal symptoms within
24 hours of taking sildenafil for erectile dysfunction

Key Concept/Objective: Understand the appropriate management of the symptoms of chronic stable angina

Beta blockers are generally well tolerated in patients with chronic obstructive pulmonary disease, although they may exacerbate bronchospasm in patients with severe asthma. Calcium channel blockers can be used as monotherapy in the treatment of chronic stable angina, although combinations of beta blockers and calcium channel blockers relieve angina more effectively than either agent alone. Calcium channel blockers are contraindicated in the presence of decompensated congestive heart failure, although the vasoselective dihydropyridine agents amlodipine and felodipine are tolerated in patients with clinically stable left ventricular dysfunction. Nitroglycerin and nitrates should not be used within 24 hours of taking sildenafil or other phosphodiesterase inhibitors used in the treatment of erectile dysfunction, because of the potential for life-threatening hypotension. (Answer: D—Nitrates should not be used as treatment for anginal symptoms within 24 hours of taking sildenafil for erectile dysfunction)

47. A 70-year-old man presents to establish care. His medical history is remarkable for hypertension and a myocardial infarction 3 years ago. His medications include aspirin, 325 mg daily; metoprolol, 100 mg twice daily; and isosorbide mononitrate, 120 mg daily. He reports that when walking more than one block he has substernal chest pressure, which is relieved by rest. He had a cardiac catheterization 2 months ago that showed a left main coronary artery stenosis of 80%, a proximal left anterior descending artery stenosis of 60%, and a 70% stenosis of the first obtuse marginal branch. The left ventricular ejection fraction (LVEF) was estimated at 45%.

Which of the following therapies would be most beneficial for this patient?
A. Continuing the patient’s current medication regimen without modification
B. Percutaneous transluminal angioplasty (PCTA)
C. Coronary artery bypass graft (CABG)
D. Enhanced external counterpulsation therapy (EECP)
E. Transmyocardial revascularization procedure (TMR)

Key Concept/Objective: To know the indications for coronary artery bypass surgery

CABG is recommended in patients with any of the following criteria: significant left main coronary artery disease, three-vessel disease (in patients with three-vessel disease, those with LVEF < 50% have the greatest survival benefit), and two-vessel disease with significant left anterior descending coronary artery involvement or abnormal LV function (LVEF
< 50%). In patients with three-vessel disease and abnormal LVEF, the survival benefit and symptom relief of CABG are superior to those of PCTA or medical therapy. EECP involves leg cuffs that inflate and deflate to augment venous return. This therapy may be helpful in decreasing angina in patients who have refractory angina and are not candidates for revascularization. In TMR, a laser is used to create channels in the myocardium to relieve angina. This procedure has been shown to improve severe refractory angina in patients who could not be treated with conventional revascularization techniques (PCTA or CABG). For the patient described here, CABG is the preferred procedure. (Answer: C—Coronary artery bypass graft [CABG])

48. A 65-year-old woman presents to the emergency department with anterior chest pain that has been radiating to her left arm for the past 10 minutes. She had just run one block to catch a bus before she called the paramedics. Her pain was quickly relieved by two sublingual nitroglycerin tablets given by the paramedics. She said she has had similar pain with exertion over the past 3 years and has been using her husband’s nitroglycerin occasionally. Her medical history is remarkable for diabetes. Her only medication is glyburide, 5 mg daily. Her blood pressure is 110/60 mm Hg; pulse, 80 beats/min; and respirations, 20 breaths/min. Examination reveals a moderately obese woman in no apparent distress. Heart rate and rhythm are regular, without murmur, and the lungs are clear to auscultation.

Which of the following medications should not be used to treat this patient’s angina?
A. Metoprolol extended release, 100 mg p.o., q.d.
B. Aspirin, 325 mg p.o., q.d.
C. Nifedipine, 20 mg p.o., t.i.d.
D. Isosorbide dinitrate, 10 mg p.o., t.i.d.
E. Nitroglycerin sublingual, 0.4 mg, q. 5 min, p.r.n. chest discomfort

Key Concept/Objective: To understand the agents used in the pharmacologic treatment of angina

Although not an antianginal, aspirin, 75 to 325 mg daily, should be used in all patients who have angina without specific contraindications, because it has been shown to reduce the risk of adverse cardiovascular events by 33%. Beta blockers, such as metoprolol, are the cornerstone of angina treatment because they are the only antianginals shown to reduce the risk of death and myocardial infarction. Diabetes and use of hypoglycemic medications are not contraindications to beta-blocker therapy, because there is no increase in hypoglycemic events or hypoglycemic unawareness with the use of beta blockers. Nitrates, such as isosorbide dinitrate and nitroglycerin, are effective antianginals, but they do not reduce the risk of cardiac events or death. Calcium channel blockers are effective antianginals. Short-acting agents such as immediate-release nifedipine may increase the risk of vascular events and are associated with hypotension, and therefore, they should be avoided. If calcium channel blockers are used, those agents with a long half-life or slow-release formulations should be used. (Answer: C—Nifedipine, 20 mg p.o., t.i.d.)

49. A 72-year-old man with a history of myocardial infarction 10 years ago and angina presents with complaints of recurrent chest pain, which he has been experiencing over the past 4 months. This pain is retrosternal, is brought on by exertion, and is relieved by rest. The patient has been taking aspirin, longacting diltiazem, simvastatin, atenolol, and isosorbide dinitrate at maximal doses. His blood pressure is
130/80 mm Hg; pulse, 62 beats/min; and respirations, 16 breaths/min. Physical examination is normal. ECG shows normal sinus rhythm, with left bundle branch block.

Which of the following tests would be most useful in the evaluation of this patient’s angina?
A. Exercise treadmill ECG
B. Exercise treadmill cardiac nuclear imaging
C. Exercise treadmill echocardiography
D. Dobutamine echocardiography
E. Cardiac catheterization

Key Concept/Objective: To understand the modalities available for diagnostic testing and the utility of these tests in various patients

This patient has known coronary artery disease and angina that is refractory to maximal medical management. The diagnosis of angina is firmly established with high probability because this patient has known coronary artery disease and typical symptoms. The patient’s baseline ECG has left bundle branch block, and therefore, exercise stress testing is not interpretable. Exercise treadmill cardiac nuclear imaging, exercise treadmill echocardiography, and pharmacologic stress echocardiography all have higher specificity and sensitivity than conventional exercise tolerance testing and give information about functional anatomy. However, the most useful test for this patient would be cardiac catheterization, because he has symptoms despite maximal medical management, is therefore highly likely to need revascularization, and needs to have his cardiac vascular anatomy defined with cardiac catheterization. (Answer: E—Cardiac catheterization)

50. A 60-year-old man with complaints of substernal chest pressure, brought on only by vigorous activity and relieved by rest, returns for a follow-up appointment. He takes no medications and has smoked one pack of cigarettes a day for 40 years. His blood pressure is 120/70 mm Hg; pulse, 75 beats/min; and respirations, 16. Examination reveals a thin man in no distress. Heart examination reveals a regular rhythm, with no murmurs. Jugular venous pressure is estimated at 5 cm, lungs are clear to auscultation, and extremities are without edema. The patient had an exercise treadmill thallium study that showed a small reversible defect, which prompted cardiac catheterization. This revealed a 70% stenosis of the circumflex artery. His ejection fraction was estimated at 60%. His serum LDL cholesterol is 120 mg/dl, and HDL cholesterol is 35 mg/dl.

Which of the following measures would not be appropriate in this setting?
A. Atorvastatin, 80 mg p.o., q.d.
B. Nitroglycerin, 0.4 mg sublingual, p.r.n. chest pain
C. Coronary artery bypass graft (CABG)
D. Ramipril, 10 mg p.o., q.d.
E. Atenolol, 50 mg p.o., q.d.

Key Concept/Objective: To understand the management of single-vessel and two-vessel coronary artery disease (CAD)

Patients who have oneor two-vessel CAD without significant proximal left anterior descending artery stenosis, who have mild symptoms or have not received adequate antianginal therapy, and who have a small area of reversible ischemia do not benefit from revascularization with CABG or PCTA. Patients with known CAD should be treated to achieve a target LDL < 100 mg/dl. Beta blockers are effective antianginals and reduce mortality. Nitrates are useful as antianginals but do not alter mortality. Ramipril, 10 mg daily, was studied in the HOPE trial1 and was found to significantly reduce the risks of death, myocardial infarction, and stroke. The patients in that study were older than 55 years, had known cerebrovasular disease or diabetes, had one cardiovascular risk factor (e.g., smoking, hypertension, hyperlipidemia), and did not have heart failure or known low ejection fraction. (Answer: C—Coronary artery bypass graft [CABG])

1. Yusuf S, Sleight P, Pogue J, et al: Effects of an angiotensin-convertingenzyme inhibitor, ramipril, on cardiovascular events in high-risk patients: The Heart Outcomes Prevention Evaluation Study Investigators. N Engl J Med
342:145, 2000

51. A 70-year-old man presents to his physician with complaints of chest tightness. The sensation is substernal, is brought on by exertion, and is relieved by rest. He is able to walk several blocks before he notes chest pressure. His medical history is remarkable only for hypertension and hyperlipidemia. His medications are hydrochlorothiazide, 25 mg daily, and aspirin, 325 mg daily.

Which of the following statements is true for this patient regarding exercise treadmill testing?
A. It is helpful to rule out angina
B. It is helpful to establish the diagnosis of angina
C. It is helpful to either establish or exclude the diagnosis of angina
D. It is helpful to neither establish nor exclude the diagnosis of angina
E. It will not give any prognostic information about morbidity

Key Concept/Objective: To understand the pretest probability of angina and the effect of diagnostic testing on the posttest probability of angina

In this male patient, who is older than 65 years and has typical angina, the pretest probability of significant coronary atherosclerosis is 93% to 97%. Exercise treadmill ECG has limited sensitivity and specificity. Therefore, the posttest probability after a positive or a negative test is only a few percentage points different from the pretest probability in a patient such as this one, who has a high pretest probability of coronary artery disease. In this patient, then, exercise treadmill testing (ETT) is not helpful for diagnosing angina (suspicion was sufficiently high before a test was conducted), nor is it helpful in excluding the diagnosis (because a negative test would most likely be a false negative). Exercise testing is most useful for diagnosing coronary artery disease in patients with an intermediate pretest probability (e.g., 20% to 80%). The ETT does provide useful information about the severity of disease and prognosis. (Answer: D—It is helpful to neither establish nor exclude the diagnosis of angina)

For more information, see Sutton PR, Fihn SD: 1 Cardiovascular Medicine: IX Chronic Stable Angina. ACP Medicine Online (www.acpmedicine.com). Dale DC, Federman DD, Eds. WebMD Inc., New York, December 2004

Unstable Angina/Non–ST Segment Elevation MI

52. A 76-year-old woman presents to the emergency department for evaluation of nausea and mild epigastric pain that started suddenly 45 minutes ago. She denies having chest pain. Her medical history includes diabetes mellitus, hypertension, and hyperlipidemia. An electrocardiogram is interpreted as being normal.

For this patient, which of the following statements regarding the diagnosis of unstable angina is false?
A. Chest pain must be present to make a diagnosis of unstable angina
B. The physical examination is normal in many patients presenting with unstable angina
C. The initial serum troponin level is likely to be normal
D. Up to 60% of patients with unstable angina have normal ECGs
E. Serum myoglobin levels might be the first marker to be elevated in patients presenting with unstable angina

Key Concept/Objectives: To know that a significant portion of patients with unstable angina present in atypical fashion

Approximately 35% to 50% of patients with unstable angina will not have chest pain as their presenting symptom. In the National Registry of Myocardial Infarction, which included 440,000 patients, one third had atypical symptoms. In the Alabama Unstable Angina Study of Medicare beneficiaries, which included over 4,000 patients, 51.7% of patients with unstable angina had the following atypical symptoms: dyspnea (69.4%), nausea (37.7%), diaphoresis (25.2%), syncope (10.6%), arm pain (11.5%), epigastric pain (8.1%), shoulder pain (7.4%), and neck pain (5.9%). Atypical symptoms were more common in young patients (i.e., those 25 to 40 years of age), the elderly (i.e., those older than
75 years), diabetic patients, and women. In many cases, the physical examination will be normal. Troponin I and T (TnI and TnT) are cardiac-specific subunits of the thin filament–associated troponin-tropomyosin complex, which regulates striated muscle contraction. Troponins have become the primary biomarkers in the evaluation of patients with acute coronary syndrome. These markers are detected in about one third of patients without elevation in the level of creatine kinase–myocardial band. Troponins may be detectable 3 to 4 hours after the onset of ischemic symptoms; they peak at 12 to 48 hours and persist for 4 to 10 days. Myoglobin is a nonspecific biomarker found in both cardiac and skeletal muscle. It is released rapidly in response to muscle injury and is detectable 2 hours after the onset of ischemia. Interestingly, a quarter of patients diagnosed with unstable angina/non–ST segment elevation myocardial infarction (NSTEMI) will go on to develop Q waves, and up to 60% may have a normal 12-lead ECG. (Answer: A—Chest pain must be present to make a diagnosis of unstable angina)

53. A 69-year-old woman presents to the emergency department for evaluation of dyspnea and nausea. An ECG reveals ST segment depression of 1 mm in leads II, III, and aVF. The patient is given an aspirin, a beta blocker, and nitroglycerin sublingually, and the ST segment depression resolves. A diagnosis of unstable angina is made.

For this patient, which of the following statements regarding antiplatelet therapy with a thienopyridine is false?
A. Thienopyridines irreversibly bind the adenosine diphosphate (ADP) receptor on platelets, preventing fibrinogen binding and platelet aggregation
B. The use of ticlodipine has been limited by severe neutropenia and, rarely, thrombotic thrombocytopenic purpura
C. Clopidogrel should be stopped 5 to 7 days before the patient undergoes elective surgical revascularization
D. In patients undergoing percutaneous coronary intervention (PCI), guidelines recommend indefinite use of clopidogrel
E. Pretreatment with clopidogrel before PCI should be initiated more than 6 hours before the procedure

Key Concept/Objectives: To understand the use of thienopyridines in unstable angina and as an adjunct in PCI

Thienopyridines irreversibly bind the ADP receptor on platelets, preventing fibrinogen binding and platelet aggregation. In the Clopidogrel for the Reduction of Events During Observation (CREDO) study, benefit was observed for patients who received combination therapy with aspirin and clopidogrel in a bolus load administered 6 to 24 hours before PCI. The principal adverse reaction that has limited the clinical use of ticlopidine is severe neutropenia and, rarely, thrombotic thrombocytopenic purpura, which may occur within the first 3 months of therapy. An important issue with clopidogrel is bleeding during surgery; for this reason, surgery should be avoided for 5 and preferably 7 days after the last dose, because of the prolonged duration of action. Patients with unstable angina/NSTEMI for whom a percutaneous coronary intervention is planned and who are not at high risk for bleeding should receive clopidogrel for at least 1 month and for up to 9 months. (Answer: D—In patients undergoing percutaneous coronary intervention [PCI], guidelines recommend indefinite use of clopidogrel)

For more information, see Kupersmith J, Raval A: 1 Cardiovascular Medicine: X Unstable Angina and Non–ST Segment Elevation Myocardial Infarction. ACP Medicine Online (www.acpmedicine.com). Dale DC, Federman DD, Eds. WebMD Inc., New York, February 2004

Diseases of the Aorta

54. A 68-year-old woman presents to your office for a routine follow-up visit. She has no complaints. Her medical history is notable for type 2 (non–insulin-dependent) diabetes mellitus and hypertension. She has a 60-pack-year history of cigarette smoking. The physical examination is notable for a 4 to 6 cm pulsatile, nontender abdominal mass. The patient denies having abdominal pain, and she has no family history of abdominal aortic aneurysm.

Which of the following is the best step to take next in the workup of this patient?
A. The patient should return every 3 months for serial physical examinations to follow the suspected aneurysm
B. The patient should be immediately referred for surgical evaluation
C. An abdominal ultrasound should be ordered to evaluate the possible aneurysm
D. No further intervention is necessary

Key Concept/Objective: To understand the importance of early detection of abdominal aortic aneurysms

Early recognition of abdominal aortic aneurysms can be lifesaving. Most abdominal aortic aneurysms produce no symptoms and are discovered during a routine physical examination or as a result of noninvasive screening. Periods of rapid expansion or impending rupture are often marked by severe discomfort in the lower abdomen or back; the pain may radiate to the buttocks, groin, or legs. Patients with impending or actual rupture must be managed as a surgical emergency in a manner similar to that of patients with major trauma. The fact that this patient is completely asymptomatic makes simple aortic imaging a more reasonable first step than urgent surgical referral. Current recommendations are for noninvasive screening of patients of appropriate age, which is typically defined as older than 65 years but younger if there is a significant family history of or risk factors for aneurysms. Abdominal ultrasonography is the most frequently used and most practical method. Ultrasonography has a sensitivity of nearly 100% for diagnosing aneurysms of significant size and can discriminate size to within ± 3 mm. Assessment by physical examination alone is unreliable; therefore, an imaging study is needed to confirm the diagnosis of aortic aneurysm and, if present, to determine the size. Aneurysms larger than 6 cm in diameter are generally referred for surgery; those less than 4 cm in diameter are generally watched. (Answer: C—An abdominal ultrasound should be ordered to evaluate the possible aneurysm)

55. A 62-year-old man with long-standing hypertension presents to your office for evaluation of a nonproductive cough. He has had the cough for 2 or 3 months, and it is getting progressively worse. He is without other complaint. The patient’s blood pressure is 168/94 mm Hg; other vital signs are unremarkable. A chest x-ray reveals a widening of the mediastinum. Spiral CT reveals a thoracic aortic aneurysm with impingement upon the trachea.

Which of the following statements regarding thoracic aortic aneurysms is true?
A. Aneurysms that are invading local structures should usually be resected
B. In contrast to abdominal aortic aneurysms, the size of a thoracic aortic aneurysm is not a critical issue in terms of risk of rupture
C. In contrast to abdominal aortic aneurysms, careful control of blood pressure is not a crucial issue for patients with thoracic aortic aneurysms
D. Most thoracic centers recommend surgery for thoracic aortic aneurysms that exceed 7 cm in otherwise suitable surgical candidates

Key Concept/Objective: To understand the importance of surgical referral for symptomatic thoracic aortic aneurysms

This patient has a cough secondary to impingement by a thoracic aortic aneurysm on his trachea. The fact that his cough is a new problem likely reflects expansion of his aneurysm. Aneurysms that are invading local structures or creating a marked vascular effect should usually be resected. Because size is a critical issue in terms of the risk of rupture, the initial size and potential growth of an aneurysm are important factors in the decision whether to operate on asymptomatic aneurysms. Careful control of blood pressure is crucial for all patients and may require medical therapy, particularly with beta blockers, which may also slow the rate of aneurysm growth. Currently, most thoracic centers recommend surgery for aneurysms that exceed 5.5 to 6 cm in an otherwise suitable surgical candidate. (Answer: A—Aneurysms that are invading local structures should usually be resected)

56. A 75-year-old male patient is seen in the emergency department with a severe midscapular pain. He has been experiencing this pain for 3 hours. His pain does not radiate, and he states that it has a “tearing quality.” He has no shortness of breath, anterior chest pain, or change in pain with change in position. He denies having a personal or family history of heart disease, and he denies any history of chest pain. Current vital signs are unremarkable, with the exception of his blood pressure, which is 190/105 mm Hg. His pain does not respond to sublingual nitroglycerin and intravenous beta blockade. There is no difference in pulse or blood pressure between the upper extremities. Cardiac examination is normal, with the exception of a loud fourth heart sound. Electrocardiography reveals nonspecific ST-T wave changes, along with findings consistent with left ventricular hypertrophy. A chest x-ray reveals a widening of the mediastinum and a large cardiac silhouette without pleural effusions.

Which of the following is the most appropriate step to take next in the workup of this patient?
A. Initiate anticoagulation therapy immediately and activate the heart catheterization laboratory
B. Order a stat spiral CT to rule out thoracic aortic dissection
C. Order stat aortography to rule out thoracic aortic dissection
D. Admit the patient for serial cardiac enzyme assays and ECGs to rule out myocardial infarction

Key Concept/Objective: To understand the importance of rapid diagnosis of thoracic aortic dissection

The most typical presentation of type B dissection is onset of severe interscapular pain, which may radiate down the back toward the legs. Type B dissection is frequently accompanied by hypertension, whereas type A dissection more often occurs in the presence of normal or low blood pressure. Although myocardial infarction remains a possibility, this patient’s history and examination are consistent with aortic dissection. In lieu of the considerable pretest likelihood of aortic dissection, anticoagulation should be withheld until dissection is ruled out by spiral CT or another acceptable imaging modality. Although aortography is still used in some hospitals, it is seldom the initial test for aortic dissection. The reported false negative rate for aortography is in the range of 5% to 15%. Spiral or ultrafast CT scanning gives even greater resolution than the older scanners and has a reported sensitivity and specificity for aortic dissection that exceed 95%. Blood pressure control is also an urgent consideration. (Answer: B—Order a stat spiral CT to rule out thoracic aortic dissection)

57. An 84-year-old man comes to your office complaining of a severe left temporal headache, which he has had for the past 2 days. In addition, the patient states that over the past 2 days, he has had a low-grade fever, fatigue, and loss of appetite. Upon questioning, the patient admits to muscle weakness and jaw pain with mastication but has no visual complaint. The physical examination is within normal limits, with the exception of a tender, palpable left temporal artery. Laboratory evaluation reveals a slight elevation in the white blood cell count and a marked elevation in the erythrocyte sedimentation rate.

Which of the following statements regarding giant cell arteritis is true?
A. Giant cell arteritis often affects the branches of the proximal aorta
B. Giant cell arteritis commonly occurs in patients 50 years of age or younger
C. Giant cell arteritis never results in complete blindness despite the high frequency of visual complaints
D. Standard therapy for this arteritis is prednisone, 5 to 15 mg/day

Key Concept/Objective: To recognize that giant cell arteritis affects the branches of the proximal aorta

Giant cell arteritis often affects the branches of the proximal aorta, particularly the branches supplying the head and neck, the extracranial structures (including the temporal arteries), and the upper extremities. Aortic involvement often coexists with temporal arteritis and polymyalgia rheumatica. This illness is more commonly seen in patients older than 50 years (the mean age at onset of disease is 67 years). A serious complication of this syndrome is blindness, which results when arteritis affects the ophthalmic artery. Progression to total blindness may be rapid. Visual symptoms of some type occur in as many as 50% of patients. Standard therapy for giant cell arteritis is high-dose glucocorticoid therapy (e.g., prednisone, 40 to 60 mg/day). (Answer: A—Giant cell arteritis often affects the branches of the proximal aorta)

58. A 68-year-old man with a long history of cigarette smoking presents for routine evaluation. On physical examination, he has a pulsatile abdominal mass. An ultrasound shows a 4.5 cm infrarenal aortic aneurysm. He reports no symptoms of abdominal pain or back pain.

What would you recommend for this patient?
A. Referral for surgical repair
B. Aortogram for further evaluation of aneurysm
C. Repeat ultrasound in 6 to 12 months
D. Treatment with a beta blocker

Key Concept/Objective: To understand the approach to the treatment of abdominal aortic aneurysms

Studies have shown that the likelihood of rupture is highest in patients with symptomatic or large aneurysms. Aneurysms smaller than 4 cm in diameter have a low risk (< 2%) of rupture. Aneurysms exceeding 10 cm have a 25% risk of rupture over 2 years. Current management strategies call for identification and observation of aneurysms that are asymptomatic and sufficiently small so as not to have a high risk of rupture. The median rate of expansion is slightly less than 0.5 cm a year. Aneurysms that are expanding more rapidly are more likely to rupture than stable aneurysms. Patients with aneurysms larger than 6 cm are generally referred for surgery, whereas patients with aneurysms smaller than 4 cm generally undergo observation. Evidence of expansion, particularly if the aneurysm is larger than 5 or 5.5 cm, is often taken as an indication to operate. This patient has an asymptomatic aneurysm 4.5 cm in diameter. It would be appropriate to assess its growth with ultrasonography over the next few years to see whether it is expanding more rapidly than expected. The rate of expansion is an important variable in assessing the risk of aneurysmal rupture. If the patient becomes symptomatic at any time, urgent imaging is appropriate. Aortography carries risk of contrast exposure and of atheromatous emboli, and it offers no advantages over ultrasonography for assessing the size of aneurysms. Beta blockers are not known to reduce the risk of rupture of abdominal aortic aneurysms. (Answer: C—Repeat ultrasound in 6 to 12 months)

59. A 53-year-old man presents for evaluation of severe chest pain. The pain has been present for 2 hours and is radiating down both arms. He reports no previous similar episodes of chest pain. On examination, he is diaphoretic. His blood pressure is 160/100 mm Hg bilaterally, and his pulse is 100 beats/min. Chest and cardiac examinations are normal. An electrocardiogram shows sinus tachycardia but no ST segment changes. A chest x-ray is unremarkable.

What would you recommend as the next step in the workup of this patient?
A. Aortogram
B. Abdominal ultrasound
C. Admission to cardiac care unit to rule out myocardial infarction
D. Transesophageal echocardiography (TEE)
E. Repeat chest x-ray

Key Concept/Objective: To be able to recognize aortic dissection and to know the appropriate workup

This patient with hypertension has had a severe episode of chest pain, with the pain radiating to the arm. Aortic dissection should be strongly considered, and the patient should undergo urgent evaluation. The fact that there is no discrepancy in the blood pressure measurements in the patient’s arms or that the patient’s chest x-ray is unremarkable does not rule out the diagnosis of aortic dissection. Pulse deficits are seen in approximately 25% of patients with type A dissection and in perhaps 5% to 10% of patients with type B dissection. The chest x-ray of patients with aortic dissection typically shows widening of the mediastinal silhouette, and it may also demonstrate evidence of a pleural effusion, cardiomegaly, or congestive heart failure if severe aortic regurgitation is present. The chest xray is normal in more than 15% of cases. TEE has a high sensitivity and specificity in diagnosing aortic dissections. If it is not available on an urgent basis, a chest CT or MRI is useful in establishing a diagnosis. Aortography is seldom used as an initial test for aortic dissection. The reported false negative rate for aortography ranges from 2% to 5%. More important, aortography frequently fails to detect lesions such as intramural hematomas. In addition, it takes less time to obtain a TEE using a portable device than to move a patient to an angiography suite. (Answer: D—Transesophageal echocardiography [TEE])

60. Further testing of the patient in Question 59 reveals that he has a type B aortic dissection. He has no neurologic symptoms and his renal function is normal.

What would you recommend for this patient at this time?
A. Heparin drip
B. Aggressive treatment of hypertension with beta blockers
C. Aggressive treatment of hypertension with vasodilators
D. Surgical repair of aneurysm with Dacron prosthesis
E. Surgical repair with endoprosthesis

Key Concept/Objective: To understand the treatment of type B aortic aneurysms

Emergency surgery is crucial for patients with type A aortic dissections. This patient has a type B dissection. Most such cases are managed medically by means of aggressive blood pressure control with beta blockers. If blood pressure is not adequately controlled after beta-blocker treatment, then vasodilators can be added to the beta blockers. Vasodilators should not be used in place of beta blockers. Surgery for type B dissection is indicated predominantly for patients with life-threatening complications requiring a surgical approach. Such conditions include ischemia of both kidneys, leading to reversible renal failure; the development of ischemic bowel; or ischemia involving an extremity. (Answer: B—Aggressive treatment of hypertension with beta blockers)

61. A 67-year-old man with type 2 diabetes and a long history of cigarette smoking develops severe exertional chest pain. Cardiac catheterization reveals three-vessel disease. Twenty-four hours later, he develops abdominal pain, painful toes, and a rash. On examination, he has purple discoloration of the second and fourth toes on his right foot and a lacy rash on both legs. Laboratory results are as follows: Hb,
13; HCT, 39; WBC, 9.0; BUN, 26; and Cr, 1.8.

What is the most likely diagnosis for this patient?
A. Contrast nephropathy
B. Aortic dissection
C. Atheromatous emboli syndrome
D. Abdominal aortic aneurysmal rupture

Key Concept/Objective: To be able to recognize atheromatous emboli syndrome

This patient experienced the onset of abdominal pain, purple toes, and livedo reticularis shortly after undergoing cardiac catheterization. These symptoms are consistent with atheromatous emboli syndrome. The patient also has renal insufficiency, a common feature of the syndrome. Contrast nephropathy is common in patients with diabetes, but that condition would not account for the cutaneous findings. Aortic dissection and rupture could cause abdominal pain and renal insufficiency, but that should not cause livedo reticularis or purple toes. The abdominal pain this patient experienced was most likely the result of pancreatitis caused by atheromatous emboli. (Answer: C—Atheromatous emboli syndrome)

For more information, see Eagle KA, Armstrong WF: 1 Cardiovascular Medicine: XII Diseases of the Aorta. ACP Medicine Online (www.acpmedicine.com). Dale DC, Federman DD, Eds. WebMD Inc., New York, November 2003

Pericardium, Cardiac Tumors, and Cardiac Trauma

62. A 24-year-old man presents to the emergency department complaining of chest pain. He reports having substernal chest pain of 2 days’ duration. The pain is worse with inspiration and is alleviated by maintaining an upright position. He also reports having had a fever recently. His medical history and physical examination are unremarkable. An ECG shows 2 mm elevation of the ST segment in precordial leads, without reciprocal changes and with concomitant PR segment depression. An echocardiogram is normal.

What is the most likely diagnosis and the most appropriate treatment approach for this patient?
A. Acute pericarditis; start a nonsteroidal anti-inflammatory drug (NSAID)
B. Acute pericarditis; start prednisone
C. Acute pericarditis; repeat echocardiogram in 1 week to confirm diagnosis
D. ST elevation myocardial infarction; start thrombolytics

Key Concept/Objective: To understand the diagnosis and treatment of acute pericarditis

The clinical diagnosis of acute pericarditis rests primarily on the findings of chest pain, pericardial friction rub, and electrocardiographic changes. The chest pain of acute pericarditis typically develops suddenly and is severe and constant over the anterior chest. In acute pericarditis, the pain worsens with inspiration—a response that helps distinguish acute pericarditis from myocardial infarction. Low-grade fever and sinus tachycardia also are usually present. A pericardial friction rub can be detected in most patients when symptoms are acute. Electrocardiographic changes are common in most forms of acute pericarditis, particularly those of an infectious etiology in which the associated inflammation in the superficial layer of myocardium is prominent. The characteristic change is an elevation in the ST segment in diffuse leads. The diffuse distribution and the absence of reciprocal ST segment depression distinguish the characteristic pattern of acute pericarditis from acute myocardial infarction. Depression of the PR segment, which reflects superficial injury of the atrial myocardium, is as frequent and specific as ST segment elevation and is often the earliest electrocardiographic manifestation. Analgesic agents, salicylates, or NSAIDs are often effective in reducing pericardial inflammation. Corticosteroids should be reserved for severe cases that are unresponsive to other therapy, because symptoms may recur after steroid withdrawal. The absence of a significant effusion on echocardiography is not evidence against acute pericarditis. (Answer: A—Acute pericarditis; start a nonsteroidal anti-inflammatory drug [NSAID])

63. A 40-year-old woman is being evaluated for fever. She started having fever 6 weeks ago. Other symptoms include an erythematous rash, fatigue, and weight loss. Her medical history is significant for hypertension. She takes hydrochlorothiazide. On physical examination, the patient’s temperature is found to be 100.6° F (38.1° C), a 3/6 murmur is noted in the mitral area, and an erythematous rash is seen on both legs. A complete blood count shows anemia; the patient’s erythrocyte sedimentation rate (ESR) is elevated at 80 mm/hr. A transthoracic echocardiogram shows a 2 cm pedunculated mass in the left atrium.

Which of the following is the most likely diagnosis for this patient?
A. Metastatic colon adenocarcinoma
B. Cardiac rhabdomyosarcoma
C. Papillary fibroelastoma
D. Cardiac myxoma

Key Concept/Objective: To be able to recognize cardiac myxomas

Cardiac tumors may be either primary or secondary and either benign or malignant. Metastatic cardiac involvement occurs 20 to 40 times more frequently than primary tumors. Eighty percent of all primary cardiac tumors are benign; myxomas account for more than half of these in adults. Myxomas consist of scattered stellate cells embedded in a mucinous matrix. They are found in the cavities of the heart, attached to the endocardial wall or heart valves by either a narrow stalk or a broader pedicle. About 70% of myxomas are in the left atrium. Myxomas are most often manifested clinically by mechanical hemodynamic effects, which often simulate mitral or tricuspid stenoses or regurgitation. Intermittent obstruction of the valve orifice can lead to syncope or to remarkable changes in physical signs that are sometimes related to changes in body position. Another manifestation is a constitutional disturbance consisting of fatigue, fever, erythematous rash, myalgias, and weight loss, accompanied by anemia and an increased ESR. The constitutional symptoms may be caused by production of interleukin-6 by the myxoma. Papillary fibroelastomas are small tumors, usually attached to cardiac valves; they can be a cause of cardioembolic stroke. Rhabdomyosarcoma is a malignant primary tumor of the heart. About 10% of patients who die of malignant disease have metastatic cardiac involvement, but the metastases produce symptoms in only 5% to 10% of the affected patients. The most frequent clinical manifestation is pericardial effusion with cardiac tamponade. Extensive solid tumor in and around the heart is less common but may resemble constrictive pericarditis or effusive-constrictive pericarditis. Invasion of the myocardium most often manifests clinically as arrhythmias; atrial flutter and atrial fibrillation are particularly common. (Answer: D—Cardiac myxoma)

64. A 16-year-old male becomes unresponsive immediately after being hit in the chest by a baseball in a local game. A family member reports that the patient has a “heart murmur.” He takes no medications. The emergency medical service is called and finds the patient to be pulseless. Resuscitation attempts are started and are unfortunately unsuccessful.

Which of the following is the most likely mechanism behind this patient’s cardiac arrest?
A. Impaired wall motion secondary to myocardial necrosis
B. Ruptured myocardium and pericardial tamponade secondary to cardiac trauma
C. Ventricular tachycardia secondary to hypertrophic cardiomyopathy
D. Ventricular fibrillation secondary to trauma during repolarization

Key Concept/Objective: To understand the mechanisms of blunt cardiac trauma

Cardiovascular injury may be either blunt (i.e., nonpenetrating) or penetrating. Automobile accidents are the most common cause of blunt cardiovascular trauma; gunshots and stabbings are the most common causes of penetrating trauma. Both types of injury can damage the myocardium, the valves, the coronary arteries, the pericardium, and the great vessels, especially the aorta. Myocardial contusion is the most common blunt injury. The right ventricle, because of its location immediately beneath the sternum, is the chamber most often involved. The pathologic changes in myocardial contusion consist of myocardial necrosis with hemorrhage, which may range in severity from scattered petechiae to intramural extravasations with associated transmural necrosis. In some instances, coronary arterial occlusion with secondary myocardial infarction is present. Seemingly innocuous blows to the chest by missiles such as baseballs or hockey pucks may cause sudden arrhythmic death, probably when they strike directly over the heart during the vulnerable portion of the T wave and induce ventricular fibrillation. The most important complication of myocardial contusion is cardiac arrhythmia. Hypotension, intracardiac thrombus, congestive heart failure, and cardiac tamponade occur occasionally. Blunt trauma may injure any of the cardiac valves and lead to valvular regurgitation. Traumatic valvular regurgitation is more likely to be recognized after the patient has recovered from the acute injury; it is less likely to play a major role in the early postinjury course. (Answer: D—Ventricular fibrillation secondary to trauma during repolarization)

65. A 23-year-old man reports a 3-day history of a constant left-sided chest pain, which worsens with inspiration and activity. His symptoms were preceded by several days of fatigue, rhinorrhea, and cough. He is worried that he has broken a rib from coughing. He reports no other symptoms and has no risk factors for cardiovascular disease. On examination, he is comfortable. Other findings on physical examination are as follows: blood pressure, 120/70 mm Hg; pulse, 94 beats/min; respiratory rate, 12 breaths/min; temperature, 100.2° F (37.9° C). His lungs are clear. Cardiovascular examination shows tachycardia, but otherwise the results are normal.

Which of the following should be the appropriate step to take next in this patient’s workup?
A. Chest x-ray
B. Complete blood count
C. Arterial blood gas test
D. Electrocardiogram
E. None of the above

Key Concept/Objective: To be able to recognize the presentation of acute benign viral pericarditis

This patient’s presentation is classic for acute viral pericarditis: constant anterior chest pain that is worse with inspiration, tachycardia, and a low-grade fever. A pericardial friction rub is often heard when patients are symptomatic but may be missed on examination. The differential diagnosis includes pneumonia, spontaneous pneumothorax, and musculoskeletal pain; an electrocardiogram would be the appropriate first step in the evaluation. A finding of diffuse ST segment elevations without reciprocal changes or PR depressions would confirm the diagnosis of viral pericarditis. (Answer: D— Electrocardiogram)

66. The patient in Question 65 is found to have PR depressions on electrocardiography.

What should be the next step in this patient’s management?

A. Hospital admission
B. Treatment with antibiotics
C. Treatment with NSAIDs
D. Treatment with prednisone
E. Treatment with codeine

Key Concept/Objective: To understand the management of acute pericarditis

This patient has acute benign pericarditis. Anti-inflammatory medications, including aspirin, are usually effective for reducing pericardial inflammation and decreasing pain. Codeine or another narcotic may be added for pain relief if needed. Although prednisone is effective as well, steroids are generally reserved for patients who are unresponsive to other treatments, because symptoms may recur after steroid withdrawal. Most acute pericarditis is viral in origin. Patients do not require hospitalization unless they have other complications such as arrhythmia or tamponade. (Answer: C—Treatment with NSAIDs)

67. A 44-year-old man on long-term dialysis for lupus nephritis presents with progressive dyspnea on exertion. He has no chest pain or lower extremity edema, nor does he have any other symptoms. His weight has remained stable. Other results of his physical examination are as follows: blood pressure, 130/70 mm Hg; pulse, 84 beats/min; respiratory rate, 14 breaths/min. His neck veins are elevated, and the elevation increases upon inspiration. His lungs are clear. His cardiovascular examination is remarkable for an extra sound in early diastole, and he has no paradoxical pulse. His hematocrit is normal, and the results of pulmonary function studies and electrocardiography are unremarkable.

What would be the definitive diagnostic workup for this patient?

A. High-resolution chest CT
B. Transthoracic echocardiogram
C. Right and left cardiac catheterization
D. Bronchoscopy
E. A and C

Key Concept/Objective: To be able to recognize constrictive pericarditis

Given this patient’s symptoms and his history of dialysis, he most likely has constrictive pericarditis. Definitive diagnosis requires demonstration of a thickened pericardium and equalization of right and left heart pressures. Findings of elevated central pressures in the absence of other signs of congestive heart failure are very helpful. In contrast to cardiac tamponade, paradoxical pulse is present, and the Kussmaul sign can occasionally be seen. (Answer: E—A and C)

For more information, see Eagle KA, Armstrong WF: 1 Cardiovascular Medicine: XII Diseases of the Aorta. ACP Medicine Online (www.acpmedicine.com). Dale DC, Federman DD, Eds. WebMD Inc., New York, November 2003

Congenital Heart Disease

68. A 26-year-old woman is being evaluated for dyspnea, which she experiences when she engages in physical activity. She has been having these symptoms for the past 4 months. She denies having chest pain, orthopnea, or paroxysmal nocturnal dyspnea. The patient’s medical history is significant for her having one episode of atrial fibrillation 1 month ago. Her physical examination shows fixed splitting of S2 and a 2/6 systolic murmur in the pulmonic area. An electrocardiogram shows mild right axis deviation and an rSR’ pattern in V1. A chest x-ray reveals an enlarged right atrium and main pulmonary artery.

Which of the following is the most likely diagnosis for this patient?
A. Atrioventricular septal defect (AVSD)
B. Ostium secundum atrial septal defect (ASD)
C. Ventricular septal defect (VSD)
D. Dextrotransposition of the great arteries

Key Concept/Objective: To be able to recognize an ASD

ASDs occur in three main locations: the region of the fossa ovalis (such defects are termed ostium secundum ASDs); the superior portion of the atrial septum (sinus venosus ASDs); and the inferior portion of the atrial septum near the tricuspid valve annulus (ostium primum ASDs). The last two are considered to be part of the spectrum of AVSDs. Ostium secundum ASDs are the most common variety, accounting for over half of ASDs. Most patients with ostium secundum ASDs are asymptomatic through young adulthood. As the patient reaches middle age, compliance of the left ventricle may decrease, increasing the magnitude of left-to-right shunting. Long-standing atrial dilatation may lead to a variety of atrial arrhythmias, including atrial fibrillation. A substantial number of middle-aged patients report dyspnea. The hallmark of the physical examination in ASD is the wide and fixed splitting of the second heart sound. A systolic murmur (from increased pulmonary flow) is common. On electrocardiography, the QRS axis is usually normal in patients with ostium secundum ASD but may be slightly rightward, and an rSR’ pattern is common in the right precordial leads. The chest x-ray reveals an enlarged right atrium, right ventricle, and main pulmonary artery. The diagnosis is confirmed by echocardiography. AVSDs include a complex spectrum of disorders involving abnormalities of the atrioventricular septum and, frequently, the atrioventricular valves. Patients with AVSDs can present with symptoms and physical findings similar to patients with ostium secundum ASD. An additional pansystolic murmur can be found in patients with a complete AVSD. Left axis deviation is present in the majority of patients with AVSD; in contrast, right axis deviation is found in patients with ostium secundum ASD. The classic physical finding of a VSD is a harsh pansystolic murmur, heard best at the left lower sternal border. Electrocardiography may be normal or show evidence of left ventricular hypertrophy and a pattern of diastolic overload. Dextrotransposition of the great arteries is a cyanotic congenital cardiopathy. Survival beyond the first year without surgical repair is uncommon. (Answer: B—Ostium secundum atrial septal defect [ASD])

69. A 35-year-old man presents to a hospital with fatigue and fever of 3 weeks’ duration. He denies having chest pain, dyspnea, or orthopnea. When giving his medical history, he reports that he has had a “heart murmur” since birth. On physical examination, the patient is found to have a temperature of 101° F (38.3° C); a 3/6 harsh murmur is heard in the left lower sternal border; and the presence of small, tender nodules are noted on two fingers. These are the only abnormal findings on physical examination.

Which of the following cardiac anomalies is most consistent with this patient’s clinical presentation?
A. Ostium primum ASD
B. Patent ductus arteriosus (PDA)
C. Eisenmenger syndrome
D. VSD

Key Concept/Objective: To be able to recognize VSD

This patient has had an asymptomatic heart murmur for a long time, and he now presents with symptoms and signs consistent with infectious endocarditis. VSDs are among the most common congenital cardiac disorders seen at birth but are less frequently seen as isolated lesions in adulthood. This is because most VSDs in infants either are large and lead to heart failure, necessitating early surgical closure, or are small and close spontaneously. With the exception of patients who contract infective endocarditis or those with Eisenmenger syndrome, adults with VSD are asymptomatic. The classic physical finding of VSD is a harsh, frequently palpable, pansystolic murmur heard best at the left lower sternal border. Echocardiography is the procedure of choice for determining the location, size, and hemodynamic significance of a VSD. ASDs are characterized by the presence of wide and fixed splitting of the second heart sound. The murmur seen in PDA is a continuous murmur. Eisenmenger syndrome is a serious complication of long-standing left-to-right shunts, in which severe, irreversible pulmonary hypertension develops. The presence of cyanosis is characteristic; symptoms such as dyspnea and chest discomfort can be seen. (Answer: D—VSD)

70. A 40-year-old man comes to your office as a new patient to establish primary care. He is asymptomatic.
His physical examination reveals an early systolic click and a 2/6 murmur in the aortic area. An ECG is normal. An echocardiogram shows a bicuspid aortic valve without significant flow obstruction. His ventricle size and function are normal.

Which of the following is the most appropriate therapeutic intervention for this patient at this time?
A. Instructions about endocarditis prophylaxis
B. Aortic valve replacement
C. Balloon valvuloplasty
D. No intervention is required

Key Concept/Objective: To understand the treatment of a bicuspid aortic valve

As much as 2% of the population have congenitally bicuspid aortic valves. A bicuspid aortic valve may present as an incidental finding on physical examination or echocardiography done for other reasons; as significant aortic stenosis (AS); as regurgitation; or as infective endocarditis. On physical examination, the cardinal sign of a bicuspid aortic valve is an early systolic ejection click. If no significant hemodynamic abnormality is present, either no murmur or a soft ejection murmur may be heard; a very mild murmur of aortic regurgitation (AR) is not uncommon, even with hemodynamically insignificant bicuspid aortic valves. AS or AR from any other cause will produce similar findings. Both the presence of a bicuspid aortic valve and its hemodynamic significance can be determined by echocardiography. Serial studies are useful in following the progression of the lesion. All patients with bicuspid aortic valves—even those with no significant stenosis or regurgitation—should be given instructions regarding endocarditis prophylaxis. Patients with AR from a bicuspid valve who are asymptomatic and have normal systolic function are followed with echocardiograms and physical examinations at regular intervals. If they begin to show decreasing systolic function, symptoms of heart failure, or progressive dilation of the left ventricle, surgical replacement of the aortic valve is indicated. (Answer: A— Instructions about endocarditis prophylaxis)

71. A 15-year-old girl is being evaluated for a heart murmur. She is asymptomatic. On physical examination, her blood pressure is 174/104 mm Hg on her right arm. Her pulses are 2+ on her upper extremities. The femoral pulses are slightly lower in amplitude than the radial pulses. Her cardiac examination reveals a short midsystolic murmur in the left infrascapular area.

For this patient, which of the following is most likely to be found on additional studies?
A. Downward displacement of the tricuspid valve annulus toward the right ventricle apex on echocardiogram
B. Stenotic pulmonic valve on echocardiogram
C. Rib notching and dilatation of the aorta on chest x-ray
D. Cardiomegaly and pulmonary engorgement on chest x-ray

Key Concept/Objective: To be able to recognize aortic coarctation

In this patient, the findings on physical examination are consistent with coarctation of the aorta. Coarctation is a common cause of secondary hypertension. Although lower-extremity claudication may occur, patients are commonly asymptomatic. The cardinal feature on physical examination is the differences in pulses and blood pressures above the coarctation as compared to below the coarctation. In coarctation of the aorta, the femoral pulse will occur later than the radial pulse, and it is often lower in amplitude. Because of variations in anatomy, blood pressure should be evaluated in both arms and in either leg when evaluating for coarctation of the aorta. When the coarctation is distal to the origin of the left subclavian artery, both arms will be in the high-pressure zone and both legs in the lowpressure zone. However, some coarctations are proximal to the left subclavian. Thus, the left arm and both legs will be in the low-pressure zone, and the diagnosis may be missed if only the left arm is used for measuring blood pressure. In addition to differential blood pressures, physical examination may also reveal a murmur across the coarctation that can be best heard in the left infrascapular area. Dilatation of the aorta proximal and distal to the coarctation site may lead to a so-called 3 sign on chest x-ray. Rib notching is often present; this term refers to apparent effacement or so-called scalloping of the lower edges of ribs because of large, high-flow intercostal collateral vessels that develop as a compensatory mechanism to bypass the narrowing at the coarctation site. In patients with Ebstein anomaly, downward displacement of the tricuspid valve annulus toward the right ventricle apex is seen on echocardiogram. (Answer: C—Rib notching and dilatation of the aorta on chest x-ray)

72. A 32-year-old man presents to establish care. His medical history is unremarkable. On physical examination, the patient’s blood pressure is 132/84 mm Hg, and his pulse is 88 beats/min. Cardiac examination reveals a soft systolic murmur, heard best at the left upper sternal border. The second heart sound is split and has no variability with respirations. There is no evidence of cyanosis.

What is the most likely diagnosis for this patient?
A. Coronary arteriovenous fistula
B. Bicuspid aortic valve
C. Ostium secundum defect
D. Tetralogy of Fallot
E. Pulmonary insufficiency

Key Concept/Objective: To be able to recognize the presentation of ASD

This patient presents with a soft systolic murmur. The remarkable finding is that of a fixedsplit second heart sound. This is seen in a number of clinical situations, related to either delayed closure of the pulmonary valve or early closure of the aortic valve. Delayed pulmonary valve can occur in ASD, right bundle branch block, pulmonary stenosis, pulmonary hypertension, and right ventricular failure. The aortic valve may close early in VSD or severe mitral insufficiency. The most common variant of the ASD is the ostium secundum. (Answer: C—Ostium secundum defect)

73. What would be the most appropriate initial diagnostic study for the patient in Question 72?
A. Left heart catheterization
B. Transthoracic echocardiography
C. Transesophageal echocardiography
D. Cardiac MRI
E. Right heart catheterization

Key Concept/Objective: To understand the appropriate evaluation of ASD

The patient presented most likely has an ASD. The most appropriate initial test would be a transthoracic echocardiogram. If this test is not helpful, one might proceed to transesophageal echocardiography. Cardiac catheterization is reserved for situations in which ventricular failure, pulmonary hypertension, or both are clinically likely. (Answer: B— Transthoracic echocardiography)

74. A 23-year-old woman with Down syndrome (trisomy 21) presents to establish care. In general, she appears comfortable. There is no evidence of cyanosis. Her cardiac examination reveals a soft systolic murmur with a midsystolic click. The clinical diagnosis of mitral valve prolapse is made. The patient’s mother wonders if there are any other cardiac issues.

Which congenital heart anomaly is this patient most likely to have?
A. Ostium primum defect
B. PDA
C. Tetralogy of Fallot
D. Pulmonary stenosis
E. Transposition of great vessels

Key Concept/Objective: To be able to recognize cardiac manifestations of Down syndrome

Patients with Down syndrome have roughly a 40% chance of congenital cardiac anomalies. The most common are the endocardial cushion defects (especially ostium primum defects). They are also at risk for AR, tetralogy of Fallot, and pulmonary hypertension. Tetralogy of Fallot is much less common than the endocardial cushion defects. Interestingly, these patients are also at increased risk for mitral valve prolapse. (Answer: A— Ostium primum defect)

75. A 54-year-old man has been referred to you by a local dentist. He states that he has been told that he has a heart murmur, and the dentist recommended evaluation before planned dental extractions. On examination, the patient is found to have a III/VI systolic murmur along the right upper sternal border. Carotid upstroke is brisk. Transthoracic echocardiography reveals a bicuspid aortic valve. No allergies are noted.

What is the most appropriate action to take next for this patient?
A. No prophylaxis is necessary
B. Prophylaxis with ceftriaxone, 1 g I.M. 1 hour before procedure
C. Prophylaxis with azithromycin, 1 g p.o. 1 hour before procedure
D. Prophylaxis with vancomycin, 1 g I.V. 1 hour before procedure
E. Prophylaxis with amoxicillin, 2 g p.o. 1 hour before procedure

Key Concept/Objective: To understand the appropriate use of prophylactic antibiotics

According to the most recent American Heart Association (AHA) recommendations, this patient is a candidate for prophylactic antibiotics, preferably amoxicillin. Most congenital cardiac malformations are deemed to be in the moderate-risk category. Complex cyanotic congenital anomalies are considered high risk (along with prosthetic valves and previous history of endocarditis). The exceptions, in which prophylaxis is not recommended, include isolated secundum ASD, surgical repairs of septal defects (both atrial and ventricular), and PDA. Refer to the AHA guidelines for full details on antibiotic choices and indications. (Answer: E—Prophylaxis with amoxicillin, 2 g p.o. 1 hour before procedure)

76. A 19-year-old woman presents with complaint of dyspnea on exertion and mild fatigue. She has no significant medical history. She does not use tobacco. She is on no regular medications. On examination, her lungs are clear. Cardiac examination reveals a II/VI systolic murmur at the second left intercostal space, which varies with inspiration.

What is the most likely diagnosis for this patient?
A. Pulmonary stenosis
B. ASD
C. VSD
D. Bicuspid aortic valve
E. Idiopathic hypertrophic subaortic stenosis

Key Concept/Objective: To be able to recognize the clinical presentation of pulmonary stenosis

This patient presents with a systolic murmur that varies with respiration. This makes it likely that the etiology is right sided. Given the location, pulmonary stenosis is more likely than tricuspid regurgitation. These murmurs vary with respiration because filling of the right heart is significantly affected by inspiration (as blood is returning from outside the chest and is therefore influenced by the negative thoracic pressure). Treatment for this disorder is most commonly done via transcatheter balloon dilatation. (Answer: A—Pulmonary stenosis)

For more information, see Mahoney LT, Skorton DJ: 1 Cardiovascular Medicine: XV Congenital Heart Disease. ACP Medicine Online (www.acpmedicine.com). Dale DC, Federman DD, Eds. WebMD Inc., New York, October 2003

Peripheral Arterial Disease

77. A 35-year-old man with a history of superficial thrombophlebitis and bronchitis reports with bilateral foot pain of two days’ duration. Over the past year, he has had several episodes of severe burning pain involving the foot arches and several toes. On a scale of 1 to 10, the severity of the pain is 7 to 9, and the pain persists both at rest and with ambulation. The patient smokes one to two packs of cigarettes a day, drinks one or two beers daily, and uses no illicit drugs. On examination, he is slender; his feet are red and cold, and there are ulcerations around the margins of several toenails. The femoral pulses are intact, and the dorsalis pedis and posterior tibialis pulses are absent bilaterally. The pain is not worsened by deep palpation. Microscopic capillaroscopy is negative for dilated capillaries.

What is the most likely diagnosis for this patient?
A. Plantar fasciitis
B. Spinal stenosis
C. Raynaud phenomenon
D. Atherosclerotic claudication
E. Thromboangiitis obliterans

Key Concept/Objective: To know the features of thromboangiitis obliterans and to be able to distinguish this disorder from other diseases in the differential diagnosis of foot pain

Thromboangiitis obliterans (also called Buerger disease) causes inflammatory blockage of arterioles in the distal extremities and is seen in male smokers who are less than 40 years of age. Other typical features include a history of recurrent thrombophlebitis, rest pain, and findings of dependent rubor and an absence of distal pulses. Plantar fasciitis is usually not painful when the patient is at rest; it is exacerbated by weight bearing and deep palpation on examination and is not accompanied by loss of distal pulses. Spinal stenosis usually occurs in older patients and presents as lower extremity pain that is exacerbated by standing or walking and is relieved by rest. Atherosclerotic claudication is also seen in older patients. It follows a steadily progressive course, beginning with exercise-induced pain and progressing slowly (over months to years) to pain at rest. In addition, larger, more proximal vessels are usually affected, with corresponding exercise-induced pain in the buttocks, thighs, or calves. Raynaud phenomenon is seen mostly in women; it is caused by vasospasm of small arterioles, more often in the hands than in the feet. The vasospasm is precipitated by cold or stress and causes sequential color changes in the digits from white to blue to red. These changes in color may be accompanied by a sensation of cold, numbness, or paresthesias but usually not severe pain. Peripheral pulses usually remain intact even during episodes of vasospasm. (Answer: E—Thromboangiitis obliterans)

78. A 63-year-old woman with a history of obesity, diabetes, hypertension, hyperlipidemia, and severe hip arthritis is found to have a foot ulcer. She does not know how long it has been present but reports nocturnal foot pain of several months’ duration that improves when she dangles her foot over the edge of the bed. She denies having fever or leg swelling. On examination, an ulcer 2 cm in diameter is seen under the first metatarsal head of her left foot; the base of the ulcer is necrotic, and there is no visible granulation tissue. As the patient sits in the clinic chair, her distal extremities are seen to be a deep red, and the skin of the distal extremities is smooth and thin, without hair. She is able to detect a monofilament on sensory examination of the feet. The dorsalis pedis and posterior tibialis pulses cannot be palpated. Her ankle brachial index is 0.35.

What is the most likely cause of this patient’s foot ulcer?
A. Diabetic neuropathy
B. Venous stasis
C. Vasculitis
D. Arterial insufficiency
E. Pyoderma gangrenosum

Key Concept/Objective: To be able to recognize ulceration associated with arterial insufficiency

The findings of rest pain that worsens when the patient is in the horizontal position and an ankle brachial index of less than 0.4 suggest severe arterial insufficiency. Loss of distal pulses and trophic skin changes, such as loss of subcutaneous tissue and hair, are also suggestive of arterial insufficiency. In patients with arterial insufficiency, ulcers commonly occur on the feet, particularly in weight-bearing areas or at sites of trauma. These areas include the area under the metatarsal heads, the ends of toes and the area between the toes, and the heel. Diabetic nephropathy also can lead to ulceration in these areas, although in patients with diabetic nephropathy, the foot is usually insensate, and therefore such patients are unable to detect a monofilament on examination. Ulcers that result from venous stasis are usually associated with edema, skin thickening, and hyperpigmentation or erythema. The base of these ulcers usually contains red, bumpy granulation tissue. Vasculitic ulcers are often associated with systemic signs of disease. With vasculitic ulcers, livedo reticularis may be present on the legs and trunk; there is no loss of distal pulses, nor would vasculitis lower the ankle brachial index. Pyoderma gangrenosum is usually seen on the anterior calf. Onset is abrupt. Pyoderma gangrenosum ulcers have a violet undermined edge, a ragged heaped-up border, and a surrounding red halo. Pyoderma gangrenosum is not associated with loss of distal pulses nor with an altered ankle brachial index. (Answer: D—Arterial insufficiency)

79. A 63-year-old woman with diet-controlled diabetes, hypertension, and a 45-pack-year history of cigarette smoking has intermittent left leg claudication and exercise-induced calf pain, which occurs when she walks three to four blocks. Her ankle brachial index is 0.7.

The patient is most likely to experience which of the following adverse health outcomes in the next 5 years?
A. Myocardial infarction
B. Hypoglycemic event
C. Limb amputation
D. Ischemic limb
E. Lung cancer

Key Concept/Objective: To understand the high risk of coronary disease in patients with atherosclerotic claudication

This woman most likely has atherosclerotic peripheral arterial occlusive disease. She has multiple risk factors for this disorder (hypertension, a history of cigarette smoking, and diabetes), she experiences exercise-induced claudication, and her ankle-brachial index is low. Most patients with peripheral vascular atherosclerosis also have coronary atherosclerosis; mortality in patients with peripheral vascular disease is usually caused by myocardial infarction or stroke. This patient’s risk of myocardial infarction far outweighs her risk of developing limb ischemia or of requiring limb amputation. Although the risk of lung cancer is 10-fold higher in cigarette smokers than in nonsmokers, this patient is less still likely to develop lung cancer than myocardial infarction: annual deaths from myocardial infarction attributable to smoking are estimated at 170,000, whereas deaths from lung cancer that are attributable to smoking number 100,000. Moreover, this patient’s coronary risk factors would place her more at risk than would be indicated by these statistics. Because this patient does not use hypoglycemic agents, she is unlikely to experience hypoglycemia. Although 2% to 4% of patients with intermittent claudication develop critical limb ischemia annually, death and morbidity from myocardial infarction are much more likely. (Answer: A—Myocardial infarction)

80. A 45-year-old woman is receiving enoxaparin and warfarin for deep vein thrombosis (DVT) of the right thigh, which developed after she underwent an abdominal hysterectomy 3 weeks ago. On day 5 of treatment, she reports abrupt onset of pain in her left leg. On examination, her blood pressure is 150/90 mm Hg; she has a regular heart rate of 95 beats/min without murmur; and she has lower extremity petechiae. Her left foot is pale, pulseless, and cold, and there is an absence of sensation. Results of laboratory testing are as follows: prothrombin time, 45; INR for prothrombin time, 2.0; platelets, 15; and hematocrit, 36.

Which of the following changes in this patient’s medication regimen should be made next?
A. Increase the heparin dose
B. Increase the warfarin dose
C. Administer tissue plasminogen activator
D. Discontinue heparin therapy
E. Discontinue warfarin therapy

Key Concept/Objective: To be able to recognize heparin-induced thrombocytopenia and associated acute arterial thrombosis and to understand that heparin must be discontinued immediately in patients with this condition

This patient is experiencing an acute arterial occlusion. Given her heparin use and her low platelet count, heparin-induced thrombocytopenia is the likely diagnosis. Discontinuance of heparin therapy as soon as possible is key in reversing this antibody-mediated process. Increasing the heparin dose or even continued exposure to low doses of heparin (as through heparin I.V. catheter flushes) will worsen, not improve, the arterial clotting and thrombocytopenia. Although therapy with catheter-directed tissue plasminogen activator (t-PA) is used for acute arterial occlusion in many cases, this patient’s recent abdominal surgery is an absolute contraindication to t-PA therapy. This patient’s low platelet count and her use of oral warfarin are relative contraindications to the use of thrombolytic therapy. She needs continued anticoagulation for her DVT and new arterial thrombus; therefore, warfarin should be continued at its currently therapeutic dosage. (Answer: D— Discontinue heparin therapy)

For more information, see Creager MA: 1 Cardiovascular Medicine: XVI Peripheral Arterial Disease. ACP Medicine Online (www.acpmedicine.com). Dale DC, Federman DD, Eds. WebMD Inc., New York, July 2001

Venous Thromboembolism

81. A 44-year-old man presents to your office complaining of right leg pain and swelling of 3 days’ duration. The patient was well until he had a wreck while riding his dirt bike 1 week ago. The patient states that he injured his right leg in this accident. Initially, his leg was moderately sore on weight bearing, but swelling and persistent pain have now developed. On physical examination, you note an extensive bruise on the patient’s right calf and 2+ edema from the foot to the midthigh. You suspect trauma-associated deep vein thrombosis (DVT).

Which of the following statements regarding DVT is true?
A. Thrombi confined to the calf are large and typically result in pulmonary venous thromboembolism (VTE)
B. The postthrombotic syndrome is a rare sequela of DVT and is associated with low morbidity
C. Most patients presenting with a new DVT have an underlying inherited thrombophilia
D. The most common cause of inherited thrombophilia associated with this illness is activated protein C resistance (factor V Leiden)

Key Concept/Objective: To understand the general features of DVT

Seventy percent of patients with symptomatic pulmonary embolism have DVT, which is usually clinically silent. Thrombi confined to calf veins are usually small and are rarely associated with pulmonary embolism. An inherited thrombophilic defect known as activated protein C resistance, or factor V Leiden, has now been established as the most common cause of inherited thrombophilia, occurring in about 5% of whites who do not have a family history of venous thrombosis and in about 20% of patients with a first episode of venous thrombosis. The second most common thrombophilic defect is a mutation (G20210A) in the 3’ untranslated region of the prothrombin gene that results in about a
25% increase in prothrombin levels. This mutation is found in about 2% of whites who have no family history of venous thrombosis and in about 5% of patients with a first episode of venous thrombosis. Elevated levels of clotting factors VIII and XI and of homocysteine also predispose patients to thrombosis. Although inherited thrombophilia is a well-described and important cause of venous thrombosis, the large majority of patients with venous thrombosis do not have an inherited thrombophilia. The postthrombotic syndrome occurs as a long-term complication in about 25% (and is severe in about 10%) of patients with symptomatic proximal vein thrombosis in the 8 years after the acute event, with most cases developing within 2 years. Clinically, the postthrombotic syndrome may mimic acute venous thrombosis but typically presents as chronic leg pain that is associated with edema and that worsens at the end of the day. Some patients also have stasis pigmentation, induration, and skin ulceration; a smaller number of patients have venous claudication on walking, caused by persistent obstruction of the iliac veins. (Answer: D— The most common cause of inherited thrombophilia associated with this illness is activated protein C resistance [factor V Leiden])

82. A 43-year-old man presents to the emergency department complaining of chest pain of 2 hours’ duration. The patient denies having any dyspnea. He has no significant medical history, nor does he have a family history of early coronary artery disease. He is a nonsmoker and an avid jogger. His chest pain is constant, is pleuritic, and does not radiate. His chest x-ray is clear, and his ECG reveals only sinus tachycardia. Blood gas measurements reveal a partial pressure of oxygen (PO2) of 58 mm Hg with a widened alveolar-arterial difference in oxygen (A-aDO2). Helical CT reveals segmental and subsegmental filling defects in the right lung.

Which of the following statements regarding anticoagulation and thrombolysis for thromboembolism is true?
A. When using unfractionated heparin, the therapeutic range for the activated partial thromboplastin time (aPTT) is 2.5 to 3.5 times the normal value
B. Low-molecular-weight heparin (LMWH) is safe and effective for the treatment of pulmonary thromboembolism
C. Because of a delay in achieving a therapeutic INR with lower doses, a starting dose of warfarin should be no less than 10 mg
D. In contrast to other thrombolytic agents, recombinant tissue plasminogen activator (rt-PA) stimulates antibody production and can induce allergic reactions

Key Concept/Objective: To understand individual therapies for pulmonary thromboembolism

When using unfractionated heparin, the dose should be adjusted as necessary to achieve a therapeutic range, which for many aPTT reagents corresponds to an aPTT ratio of 1.5 to 2.5. The published research on LMWHs, which includes over 3,000 patients treated with either once-daily or twice-daily subcutaneous doses, has established this class of anticoagulants as being safe, effective, and convenient for treating venous thrombosis and pulmonary embolism. Evidence indicates that it might be safer to use a starting dose of 5 mg of warfarin because, compared with 10 mg, the 5 mg starting dose does not result in a delay in achieving a therapeutic INR and is associated with a lower incidence of supratherapeutic INR values during the first 5 days of treatment. Because streptokinase is a bacterial product, it stimulates antibody production and can prompt allergic reactions. Antistreptococcal antibodies, which are present in variable titers in most patients before streptokinase treatment, induce an amnestic response that makes repeated treatment with streptokinase difficult or impossible for a period of months or years after an initial course of treatment. (Answer: B—Low-molecular-weight heparin [LMWH] is safe and effective for the treatment of pulmonary thromboembolism)

83. A 72-year-old man presents to the hospital with a hip fracture. An orthopedist is planning surgical repair and asks you to see the patient in consultation for preoperative assessment and advice. In particular, the orthopedist asks you to assess the patient’s need for prophylaxis against venous thrombosis and to comment on the best prophylactic regimen for the patient.

Which of the following statements regarding primary prophylaxis against venous thrombosis and thromboembolism is true?
A. Oral anticoagulation is the method of choice for moderate-risk general surgical and medical patients
B. Prophylactic therapy should typically be discontinued at the time of discharge for patients who have undergone major orthopedic surgery
C. LMWH is more effective than standard low-dose heparin in patients undergoing elective hip surgery
D. For patients undergoing genitourinary, neurologic, or ocular surgery, the most appropriate method of prophylaxis is oral anticoagulation

Key Concept/Objective: To know the correct methods of prophylaxis for venous thrombosis and thromboembolism in medical and surgical patients

Low-dose-heparin prophylaxis is the method of choice for moderate-risk general surgical and medical patients. It reduces the risk of VTE by 50% to 70% and is simple, inexpensive, convenient, and safe. Extended prophylaxis with LWMH or warfarin for an additional 3 weeks after hospital discharge should be considered after major orthopedic surgery. Extended prophylaxis is strongly recommended for high-risk patients (e.g., those with previous VTE or active cancer). LMWH is more effective than standard low-dose heparin in general surgical patients, patients undergoing elective hip surgery, and patients with stroke or spinal injury. For those undergoing genitourinary, neurologic, or ocular surgery, intermittent pneumatic compression, with or without graduated compression stockings, is effective prophylaxis against venous thrombosis and does not increase the risk of bleeding. (Answer: C—LMWH is more effective than standard low-dose heparin in patients undergoing elective hip surgery)

84. An 80-year-old patient of yours is scheduled to undergo total knee replacement. He is in excellent health, and except for osteoarthritis, his medical history is not significant. The orthopedic surgeon asks you for advice regarding VTE prophylaxis.

What would you advise for this patient?
A. LMWH is contraindicated because of the risk of bleeding; intermittent pneumatic compression devices would be preferable
B. Intermittent pneumatic compression devices are contraindicated because of the location of the surgery; LMWH is preferable
C. Aspirin, 325 mg q.d., should be started immediately after surgery
D. LMWH and intermittent pneumatic compression devices are equally effective in preventing VTE after knee surgery
E. The risk of VTE after knee replacement is so low as to make prophylaxis unnecessary

Key Concept/Objective: To know the prophylaxis for DVT after knee-replacement surgery

LMWH and intermittent pneumatic compression devices have been shown to be equally effective in preventing DVT after knee-replacement surgery. Aspirin has been shown to decrease the risk of DVT after hip fracture, but its efficacy relative to LMWH or intermittent pneumatic compression devices has never been studied, and the standard of care for postoperative DVT prophylaxis in North America does not call for its use. After kneereplacement surgery, the risk of postoperative DVT is 10% to 20%, and the rate of fatal pulmonary embolism is 1% to 5%, so prophylaxis is indicated. Prophylaxis is also cost-effective. (Answer: D—LMWH and intermittent pneumatic compression devices are equally effective in preventing VTE after knee surgery)

85. A 28-year-old woman who is 18 weeks pregnant and is G1P0 is referred to you by her obstetrician for advice regarding management of a possible VTE diathesis. Although she has no personal history of VTE, she reports that her mother and a cousin both had blood clots during pregnancy; she does not know whether they were tested for clotting disorders. She is feeling well, and her physical examination is remarkable only for her pregnancy.

Which of the following actions would you take for this patient?
A. Educate her about the symptoms of VTE and advise her to seek care immediately if she notes one of them; otherwise, no further testing or treatment is necessary
B. Start warfarin therapy with a target INR of 2 to 3
C. Start LMWH, 100 anti-10a U/kg subcutaneously q.d.
D. Start aspirin therapy, 325 mg q.d.
E. Test for antithrombin-III deficiency; if she has the deficiency, start LMWH therapy

Key Concept/Objective: To understand the management of inherited thrombophilias in pregnancy

It is possible that this woman has an inherited thrombophilia. Pregnant women with antithrombin-III deficiency have an especially high rate of VTE. If this patient tests positive, she should receive prophylactic anticoagulation therapy throughout the rest of her pregnancy. The benefits of prophylactic anticoagulation in pregnant women with protein C or protein S deficiency outweigh the risks only if they have a history of VTE. Because this patient has never had VTE, the results of testing her for these disorders would not lead to a change in management. In pregnant women with factor V Leiden mutation or G20210A prothrombin mutation, no anticoagulation therapy is recommended unless they develop a clot during the current pregnancy. In any case, LMWH is preferable to warfarin therapy because of the teratogenic effects of warfarin. Aspirin does not provide effective anticoagulation therapy for VTE. Although this woman should be educated about the signs and symptoms of VTE, this alone is not sufficient. (Answer: E—Test for antithrombin-III deficiency; if she has the deficiency, start LMWH therapy)

For more information, see Kearon C, Hirsch J: 1 Cardiovascular Medicine: XVIII Venous Thromboembolism. ACP Medicine Online (www.acpmedicine.com). Dale DC, Federman DD, Eds. WebMD Inc., New York, December 2003